nclex prep

अब Quizwiz के साथ अपने होमवर्क और परीक्षाओं को एस करें!

A client is to begin taking alendronate. Which of these instructions should the nurse emphasize when teaching about this medication? "You may take this medication after any meal, at the same time every day." "Take the medication with a full glass of milk two hours after meals." "It is recommended that you take this medication with calcium and a glass of juice." "Be sure to take this medication on an empty stomach."

"Be sure to take this medication on an empty stomach." -alendronate is used to tx + prevent osteoporosis -should be taken 1st thing in the morning w/ 6-8 oz of plain water @ least 30 minutes before other meds or foods

The nurse is providing preprocedural education to the client preparing for a barium enema. What statement made by the client indicates a need for further education? "A barium enema is used to examine the upper and lower GI tracts." "I will need to drink plenty of fluids and eat foods high in fiber after the procedure." "I will not eat or drink anything after midnight before the procedure." "I will use the prescribed laxative before the procedure."

"A barium enema is used to examine the upper and lower GI tracts." -involves filling large intestine (lower GI tract) w/ diluted barium fluid while xray images taken

A mother of a burned child asks the nurse to clarify what is meant by a third-degree burn. The best response by the nurse should include which point? "The skin layers are swollen and reddened." "All layers of the skin were destroyed in the burn." "The top layer of the skin is destroyed." "Muscle, tissue and bone have been injured."

"All layers of the skin were destroyed in the burn." -full thickness injury to dermis, epidermis + subq

A client on warfarin therapy after coronary artery stent placement calls the clinic to ask: "Can I take Alka-Seltzer for an upset stomach?" What is the best response by the nurse? "Take Alka-Seltzer at a different time of day than you take the warfarin (Coumadin)." "Select another antacid that does not inactivate warfarin (Coumadin)." Incorrect "Avoid Alka-Seltzer because it contains aspirin." "Use about half the recommended dose of Alka-Seltzer."

"Avoid Alka-Seltzer because it contains aspirin." -alka seltzer is an OTCH aspirin-antacid combo

A man diagnosed with epididymitis two days ago calls the nurse at a public health clinic to discuss his diagnosis. Which information is most important for the nurse to ask about at this time? "Did your provider recommend that you be tested for Chlamydia?" "What are you taking for pain and does it provide total relief?" "Did you know that a consequence of epididymitis is infertility?" "Do you have any questions about your care?"

"Did your provider recommend that you be tested for Chlamydia?" -epididymitis can result from chlamydia -determination of the reason for the pt's referral is important to start w/

A client is prescribed trimethoprim/sulfamethoxazole for recurrent urinary tract infections. Which information would be important to reinforce during client teaching? "Drink at least eight glasses of water a day." "Stop the medication after five days." "It is safe to take with oral contraceptives." "Be sure to take the medication with food."

"Drink at least eight glasses of water a day." -to lower risk of developing kidney stones

A client asks the nurse about including her 2 year-old and 12 year-old sons in the care of their newborn sister. Which response is an appropriate initial statement by the nurse? "Tell each child what he can do to help with the baby." "Focus on your sons' needs during the first days at home." "Suggest that your partner spend more time with the boys." "Ask the children what they would like to do for the newborn."

"Focus on your sons' needs during the first days at home." -in an expanded family, it is important for parents to reassure older children that they are loved + as important as newborn

The nurse directs an unlicensed assistive personnel (UAP) to assist the elderly surgical client to ambulate in the hallway. Which instruction by the nurse is best? "If the client is dizzy on standing, ask the client to take a few deep breaths before taking a step." "Don't forget to ask the client to put on his slippers before going for a walk." "After you assist the client back into bed, let me know how the client feels." "Have the client sit on the side of the bed for at least two minutes before helping the client to stand."

"Have the client sit on the side of the bed for at least two minutes before helping the client to stand." -nurse should give clear instructions to the UAP about what is expected for pt safety -UAP can report how far pt walked

Which statement by an older adult with chronic obstructive lung disease (COPD) indicates an understanding of the major reason to use pursed-lip breathing for episodes of dyspnea? "My mouth doesn't get as dry when I breathe with pursed lips." "I can breathe better using pursed-lip breathing because less air will be trapped in my chest." "I can breathe better when I pucker up my lips because I can control how fast I breathe in and out." "This position of my lips helps to keep my lungs open."

"I can breathe better using pursed-lip breathing because less air will be trapped in my chest." -pt/s w/ COPD have difficulty exhaling fully as a result of air trapping in the alveoli d/t weakened alveolar walls from the dx process

A 19 year-old client is paralyzed in a car accident. Which statement would indicate that the client is using the mechanism of "suppression"? "It's all the other guy's fault! He was going too fast." "I'd rather not talk about it right now." "My mother is heartbroken about this." "I don't remember anything about what happened to me."

"I don't remember anything about what happened to me." -suppression is willfully putting an unacceptable thought or feeling out of one's mind -"voluntary forgetting"

During initial evening rounds, the nurse notices a foul smell in the room of a client diagnosed with pneumococcal pneumonia who was started on intravenous antibiotics 10 hours ago. The client makes all of these statements during their conversation. Which one would alert the nurse to a potential complication of this diagnosis? "I feel hot off and on, especially when I lie in bed." "I have a sharp pain in my chest when I take a breath." "I have been sweating off and on all day" "I have been coughing up foul-tasting, brown, thick sputum."

"I have been coughing up foul-tasting, brown, thick sputum." -signals possible development of a lung abscess, a complication of pneumonia, particularly in aspiration pneumonia -sharp chest pain on inspiration (pleuritic pain) is an expected finding

A 54 year-old female explains to the health care provider that she experiences approximately 10 vasomotor symptoms of menopause ("hot flashes") throughout the day and night. Different treatment options are discussed. Which statement by the client indicates she needs further instruction from the nurse? "I should avoid spicy foods, alcohol, and caffeine." "I may need to take estrogen and progesterone for many years." "I can use a fan at home and in the workplace." "I will take gabapentin ER at bedtime."

"I may need to take estrogen and progesterone for many years." -risk of breast CA, heart attack + blood clots w/ menopausal hormone therapy -should be short term tx

A client diagnosed of bipolar disorder has been referred to a local halfway house for placement consideration. How should the nurse respond when the social worker calls to obtain information about the client's mental status and adjustment during hospitalization? "I can never give any information out by telephone. How do I know who you are?" "I am sorry. Referral information can only be provided by the client's providers." "I need to get the client's written consent before I release any information to you." "Since this is a referral, I can give you a limited amount of information."

"I need to get the client's written consent before I release any information to you."

A client is being discharged with a prescription for an iron supplement. What statement indicates a need for further teaching by the nurse? "I will have greenish-black stools from the medication." "I should not take antacids with my iron supplement." "I should take vitamin C with the iron supplement." "I should take the iron supplement with a full glass of milk."

"I should take the iron supplement with a full glass of milk." -iron should be taken w/ vitamin c or oj b/c this increases the absorption of the med -milk decreases absorption -iron causes stool to turn greenish-black

A 14 year-old with a history of sickle cell disease is admitted to the hospital with a diagnosis of vaso-occlusive crisis. Which statement by the client would most likely indicate the cause of this crisis? "I used cold medicine last week and I have gotten worse." "I knew this would happen. I've been eating too much red meat lately." "I have really been working hard practicing with the debate team at school." "I really enjoyed my fishing trip yesterday. I caught two fish."

"I used cold medicine last week and I have gotten worse." -any condition that increases the body's need for O2 or alters the transport of O2 including infection, dehydration or even cold weather may result in sickle cell crisis

The nurse is teaching diet restrictions to a client diagnosed with Addison's disease. The client indicates an understanding of the dietary restrictions when making which of these statements? "I will increase fluids and restrict sodium and potassium." "I will increase potassium and sodium and restrict fluids." "I will increase sodium, potassium and fluids." "I will increase sodium and fluids and restrict potassium."

"I will increase sodium and fluids and restrict potassium." -dehydration, hypotension, hyponatremia, hyperkalemia

A client has been given a prescription for alendronate. Which of the following statements indicate the client understands how to safely take this medication? (Select all that apply.) "I will take the pill immediately preceding weight-bearing exercise." "I will stand or sit quietly for 30 minutes after taking it." "I will always eat breakfast before taking it." "I will swallow it with 8 ounces of water." "I will notify the health care provider if I have any difficulty swallowing."

"I will stand or sit quietly for 30 minutes after taking it." "I will swallow it with 8 ounces of water." -taken 1st thing in am -taken @ least 30 minutes before eating or drinking anything "I will notify the health care provider if I have any difficulty swallowing."

The nurse is teaching a group of clients who are all diagnosed with schizophrenia and are taking an atypical antipsychotic medication. What statement made by one of the clients needs to be corrected? "I know I need to be patient but I wish it didn't take so long for this medication to really start working." "I should be careful when I get out of bed because this medication can cause my blood pressure to drop." "I'm so glad that this medication won't cause any of the tremors or tics I had when I was taking my old medication." "I'll probably gain a lot of weight on this medication and I may even develop diabetes."

"I'm so glad that this medication won't cause any of the tremors or tics I had when I was taking my old medication." -still may cause some of the same s/s -risk for higher cholesterol + triglyceride levels, wt gain + dev diabetes

The nurse is assessing a client in the emergency department. Which statement suggests that the client is experiencing acute cardiac ischemia? "When I sit up the pain gets worse." "The pain is right here in my stomach area." "As I take a deep breath the pain gets worse." "I've got a pressure deep in my chest behind my breast bone."

"I've got a pressure deep in my chest behind my breast bone." -pain that gets worse w/ deep breaths may be r/t to disorder of lungs

A nurse admits a 7 year-old to the emergency department after a leg injury. The x-rays show a femur fracture near the epiphysis. The parents ask what will be the outcome of this injury. The appropriate response by the nurse should be which of these statements? "In some instances this type of injury can cause retarded bone growth." "Bone growth is stimulated in the affected leg as therapy is initiated." "The injury is expected to heal quickly because of thin periosteum." "This type of injury shows more rapid union than that of younger children."

"In some instances this type of injury can cause retarded bone growth."

At a routine health assessment, a client tells the nurse that she is planning a pregnancy in the near future. The client asks about preconception diet changes. Which of these statements made by the nurse is the best approach in this situation? "Drink a glass of milk with each meal." "Eat at least one serving of fish weekly." "Include fiber in your daily diet." "Increase your intake of green leafy vegetables."

"Increase your intake of green leafy vegetables." -source of folic acid

The mother of a 2 year-old hospitalized child asks the nurse's advice about the child's screaming every time the mother gets ready to leave the hospital room. What is the best response by the nurse? "You might want to "sneak out" of the room once the child falls asleep." "Oh, that behavior will stop in a few days with patience from you." "I think you or your partner needs to stay with the child while in the hospital." "Keep in mind that for the age this is a normal response to being in the hospital."

"Keep in mind that for the age this is a normal response to being in the hospital." -separation anxiety stage normal for a child this age

A client who is newly diagnosed with hypertension is prescribed benazepril. What is the most important point to make when teaching the client about this medication? "Monitor your blood pressure and pulse regularly." "Take medication as directed at the same time each day, even if you feel well." "Call your health care provider if you develop a dry cough." "Notify the health care provider if there is a change in your voice."

"Notify the health care provider if there is a change in your voice." -could indicate angioedema - a potentially fatal condition

A client comes into the community health center upset and crying, stating: "I will die of cancer now that I have this disease." The client hands the nurse a piece of paper with the word "pheochromocytoma" written on it. What would be the best initial response by the nurse? "You probably have had episodes of sweating, heart pounding and headaches. Is that correct?" "Pheochromocytomas are usually noncancerous, but they do need to be treated to avoid complications." "This problem is diagnosed by blood and urine tests that reveal elevated levels of adrenaline and noradrenaline." "Computerized tomography (CT) or magnetic resonance imaging (MRI) are used to detect an adrenal tumor."

"Pheochromocytomas are usually noncancerous, but they do need to be treated to avoid complications." -best response of nurse is to address issue presented by pt, "fear of CA"

A nurse is caring for a child receiving albuterol inhaled for asthma. The parents ask the nurse why their child is receiving this medication. Which explanation by the nurse is correct? "Proventil will relax the smooth muscles in the airways." "The respiratory center in the brain that controls respirations will be stimulated." "It will decrease the swelling in the airways." "The medication is given to reduce the secretions that block the airways."

"Proventil will relax the smooth muscles in the airways." -relaxes + opens airways to make breathing easier

The nurse and a student nurse are discussing the health issues related to a laboring HBsAg-positive client. Which of these comments by the student is incorrect and indicates a need for further instruction? "The HBsAg-positive mother should not breastfeed her baby." "The infant will receive the hepatitis B immune globulin within 12 hours after birth." "The infant will receive the hepatitis B vaccine within 12 hours after birth." "The HBsAg-positive mother should be reported to the state or local health department."

"The HBsAg-positive mother should not breastfeed her baby." -not spread by breastfeeding, kissing, hugging, coughing -all persons w/ HBsAg-positive lab results should be reported to state or local health department -newborn should receive hep b immune globulin + B vaccine w/in 12 hrs after birth using different sites

The client is scheduled to have a pulmonary artery catheter (PAC) inserted. Prior to the procedure, what basic information can the nurse teach the client about a PAC? "The catheter will measure different pressures in the heart and lungs." "The catheter is inserted through the groin into the left side of the heart." "You will be unable to eat or drink anything for several hours after the procedure." "The procedure is performed under general anesthesia."

"The catheter will measure different pressures in the heart and lungs." -(Swan Ganz cath/right heart catheterization) -inserted either through groin or neck, using conscious sedation + local anesthetic @ bedside

A client is diagnosed with a spontaneous pneumothorax that requires the insertion of a chest tube with a flutter valve. What is the best explanation that the nurse should provide to the client? "The excess air from your chest will be removed by the tube." "The amount of air that enters your chest will be controlled by the flutter valve." "The tube will drain fluid from your chest." "The hole in your lung will be sealed with this tube insertion."

"The excess air from your chest will be removed by the tube." -purpose of the chest tube is to create negative pressure to allow the passive removal of the air that accumulated in the pleural space

An infant has just returned from surgery for placement of a gastrostomy tube as an initial treatment for a diagnosis of tracheoesophageal fistula (TEF). The mother asks, "When can the tube be used for feeding?" A nurse should respond with which of these comments? "Healing of the incision must be complete before feeding." "The stomach contents and air must be drained first." "Feedings can begin in five to seven days." "The feeding tube can be used immediately."

"The stomach contents and air must be drained first." -after surgery for g tube placement, the cath is left open + attached to gravity drainage for 24 hrs or >

During the well-baby checkup, the parents respond to questions about their newborn. Which of these parents' comments would reveal an initial finding that's associated with pyloric stenosis? "We notice the baby is fussy and spits up immediately after eating." "I noticed a little lump above the belly button." "The baby seems hungry all the time." "There was some spitting up at first, but now when the baby spits up, it shoots across the room."

"There was some spitting up at first, but now when the baby spits up, it shoots across the room." -forceful (projectile) vomiting is the hallmark finding of pyloric stenosis in most children -infant often hungry after vomiting + wants to feed again -olive shaped mass in upper belly

Two hours after receiving the first does of lithium, the client reports fine hand tremors. What is the nurse's best explanation for these findings? "These are common and expected side effects and should subside in a few days." "Taking lithium on an empty stomach should help minimize these symptoms." "Reducing dietary intake of sodium and fluids should minimize any side effects." "You are probably having an allergic reaction. The medication should be discontinued."

"These are common and expected side effects and should subside in a few days." -tremors are common s/e

A 2 year-old child has just been diagnosed with cystic fibrosis. The child's father asks a nurse, "What is our major concern now, and what will we have to deal with in the future?" Which statement is the best response by the nurse? "Cystic fibrosis results in nutritional concerns that can be dealt with." "Thick, sticky secretions from the lungs are a constant challenge in cystic fibrosis." "You will work with a team of experts and have access to a support group." "There is a probability of life-long complications."

"Thick, sticky secretions from the lungs are a constant challenge in cystic fibrosis." -respiratory threats are the primary concern

The nurse is examining a 2 year-old child with a tentative diagnosis of Wilm's tumor. The nurse would be most concerned about which statement by the mother? "My child has lost three pounds in the last month." "The child prefers some salty foods more than others." "All the pants have become tight around the waist." "Urinary output seems to be less over the past two days."

"Urinary output seems to be less over the past two days." -should be investigated further as it may be a sign of renal dysfunction -increased abd girth is a common finding in wilm's tumor

A 3 year-old child has findings that may suggest a neuroblastoma. While listening to the concerns of the parents, which finding is consistent with this diagnosis and requires follow-up by the health care provider? "Our child has been quieter than normal lately and has lost weight." "He seems to be getting weaker and weaker and is sometimes unsteady on his feet." "We keep having to buy him larger size pants because he's growing so big around the waist." "He doesn't seem to be going to the bathroom as much and his urine is dark yellow in color."

"We keep having to buy him larger size pants because he's growing so big around the waist." -one of the most common signs of neuroblastoma is increased abd girth d/t mass/tumor in abd

A client exhibits delusional behaviors and refuses to eat because of a belief that the food is poisoned. What should be the initial response by the nurse? "These feelings are a symptom of your illness." "You're safe here. I won't let anyone poison you." "Why do you think the food is poisoned?" "You think that someone wants to poison you?"

"You think that someone wants to poison you?" -reflective question -presents an opportunity for discussion, clarification of meaning + expressing doubt -provides verification of nurse's perceptions

A 3 year-old child has tympanostomy tubes in place. The child's parent asks the nurse if the child can swim in the family pool. How best should the nurse respond? "Your child may swim in your own pool but should not dive under the water." "Your child may swim and dive if earplugs are worn when in and around the pool." "Your child should not swim in the pool while the tubes are in place." "Your child may swim anywhere without restrictions."

"Your child may swim in your own pool but should not dive under the water." -children should also not submerge their heads under the water in the bathtub

steps for how to use a metered-dose inhaler

release med into spacer breathe in deeply remove mouthpiece from lips hold breath 10 seconds breathe out slowly

A health care provider orders digoxin 0.125 mg by mouth daily and furosemide (Lasix) 40 mg daily by mouth. Which of these foods should the nurse reinforce for the client to eat at least one serving daily? Wheat cereal Tomato juice Blueberries Pear nectar

Tomato juice

The nurse is reviewing the list of new client admissions. For which of these clients should contact precautions be implemented? A 45 year-old diagnosed with pneumonia A 60 year-old diagnosed with herpes simplex A 3 year-old diagnosed with scarlet fever A 6 year-old diagnosed with mononucleosis

A 60 year-old diagnosed with herpes simplex -d/t potentially weeping skin lesions scarlet fever = droplet mono = none

The client is a 7 year-old child with a fractured femur and extensive skin damage. Which type of traction does the nurse expect will be used? Buck's traction 90-90 traction Bryant's traction Russell traction

90-90 traction -for fx of femur

Staffing includes several registered nurses (RNs) and one licensed practice nurse (LPN). Which of these clients should the charge nurse assign to an RN? A 60 year-old client with a history of asthma and reports shortness of breath using oxygen at 2 L/min A 24 year-old post-op client newly diagnosed with type 1 diabetes mellitus, who is in the process of being discharged An 80 year-old client recovering 24 hours post right hip replacement A 56 year-old diagnosed with atrial fibrillation who recently started taking dabigatran (Pradaxa)

A 24 year-old post-op client newly diagnosed with type 1 diabetes mellitus, who is in the process of being discharged

A nurse in a well-child clinic examines many children on a daily basis. Which of these toddlers requires further follow-up? A 24 month-old who cries during examination A 13 month-old unable to walk A 20 month-old only using two- and three-word sentences A 30 month-old only drinking from a sippy cup

A 30 month-old only drinking from a sippy cup -should be able to drink from a cup w/o a cover

The nurse is performing a prekindergarten physical on a 4 year-old child and will administer a series of scheduled vaccines, including the DTaP, IPV, MMR and VAR. What information does the nurse need to know about these vaccinations? (Select all that apply.) The vaccines all contain weakened live viruses A 5/8 inch needle length is often used for subcutaneous (SubQ) injections A 20 gauge needle is used to administer the varicella (VAR) vaccine intramuscularly (IM) Either the deltoid muscle of the arm or anterolateral thigh muscle can be used The vaccines contain the preservative thimerosal Multiple immunizations should be administered a minimum of 1 inch apart

A 5/8 inch needle length is often used for subcutaneous (SubQ) injections Either the deltoid muscle of the arm or anterolateral thigh muscle can be used Multiple immunizations should be administered a minimum of 1 inch apart

The charge nurse on a cardiac step-down unit makes assignments for the team that consists of a registered nurse (RN), a licensed practical nurse (LPN), and an unlicensed assistive personnel (UAP). Which client should be assigned to the LPN? A 35 year-old who is 12 hours post cardiac catheterization A 58 year-old with hypertension, diagnosed with suspected angina A 65 year-old scheduled for discharge after angioplasty and a stent placement A 49 year-old diagnosed with a new onset atrial fibrillation with a rapid ventricular response

A 58 year-old with hypertension, diagnosed with suspected angina -those returning from surgery = RN

A nurse is assessing several clients in a long-term health care facility. Which client is at highest risk for developing a pressure ulcer? An incontinent client who has had three diarrhea stools in the past hour An 80 year-old ambulatory client with a history of diabetes mellitus A 79 year-old malnourished client on bed rest

A 79 year-old malnourished client on bed rest -weighing significantly less than ideal body wt increases number + surface area of bony prominences which are susceptible to pressure ulcers -malnutrition is a major risk factor for pressure ulcers

The nurse receives report on the following client assignments. Which client should the nurse assess first? A client who underwent a partial gastrectomy and reports feeling lightheaded A client reporting severe gastric distress after taking ibuprofen A client diagnosed with gastroesophageal reflux disease (GERD) reporting difficulty swallowing A client diagnosed with peptic ulcer disease (PUD) who reports feeling dizzy

A client diagnosed with peptic ulcer disease (PUD) who reports feeling dizzy -dizziness w/ PUD may indicate hemorrhaging difficulty swallowing can be a symptom of GERD

A nurse is assessing a 12 year-old child who has been diagnosed with hemophilia A. Which lab result would the nurse expect? An excess of red blood cells A deficiency of clotting factor VIII A deficiency of clotting factors VIII and IX An excess of white blood cells

A deficiency of clotting factor VIII

A triage nurse has four clients arrive in the emergency department within a 15-minute period. Which client should the triage nurse send back into the unit to be seen by the health care provider first? A 2 month-old infant with a history of rolling off the bed, who has bulging fontanelles and is crying An older adult client with complaints of frequent liquid brown-colored stools A teenager who got a singed beard while camping at a site that's more than an hour away A middle-aged client with intermittent pain behind the right scapula

A teenager who got a singed beard while camping at a site that's more than an hour away -pt is in the greatest danger w/ a potential of respiratory distress -any pt w/ singed facial hair has been exposed to heat or fire in close range that could have also caused damage to interior of lung

what vitamins is a child w/ CF deficient in?

A, D, K

The health care provider orders blood tests for a client diagnosed with acute hepatitis B (HBV). Which serum lab test does the nurse expect to be elevated? ALT (alanine aminotransferase) WBC (white blood cells) BUN (blood urea nitrogen) Albumin

ALT (alanine aminotransferase) -enzymes located in liver cells that can leak out into the bloodstream when liver cells injured

A pediatric clinic nurse examines a toddler with a tentative diagnosis of neuroblastoma. Which findings observed by the nurse would be associated with this problem? Abdominal mass and weakness Hearing loss and ataxia Lymphedema and nerve palsy Headaches and vomiting

Abdominal mass and weakness -irregular abd mass that crosses midline (CA normally found in adrenal glands on top of kidney) - fever, wkness, pallor, anorexia, wt loss

The client has just had an enteral feeding tube inserted. What would be the most accurate method to verify initial placement of the feeding tube? Flushing tube with saline Auscultation with air insertion Abdominal x-ray Aspiration for gastric contents

Abdominal x-ray -most objective + recommended approach -will determine if tube is in duodenum or jejunum + not in airways or lungs -after initial placement confirmed, nurse can verify placement by checking pH of aspirated gastric contents

The nurse is developing a teaching plan for parents on safety and risk-reduction in the home. Which of the following should the nurse give priority consideration to during teaching? Age of children in the home Number of children in the home Age and knowledge level of the parents Proximity to emergency services

Age of children in the home -all about safety

A nurse is assessing a 4 year-old child for possible developmental dysplasia of the right hip. Which finding should the nurse expect with this condition? Negative Barlow maneuver Abnormal gait Pelvic tip downward Positive Ortolani test

Abnormal gait -dev hip dysplasia produces a characteristic limp in children who are walking -ortolani in infants -expected clunk of test may not be present when testing older children

The parents of a child who has suddenly been hospitalized for an acute illness state that they should have taken the child to the pediatrician earlier. Which approach should the nurse use when dealing with the parents' comments? Accept their feelings without judgment Acknowledge that early care would have been better Focus on the child's needs and recovery Explain the cause of the child's illness

Accept their feelings without judgment -parents often blame themselves for their child's illness -other actions inappropriate @ this time

The nurse is caring for a client admitted to the hospital with severe left-sided flank pain and hematuria. Diagnostic tests indicate a kidney stone partially obstructing the left ureter. Which of the following outcomes is the most important for this client? Verbalizes understanding of the disease process Pain controlled with medication Tolerates diet without nausea and vomiting Adequate urinary elimination is maintained

Adequate urinary elimination is maintained -a stone that completely obstructs the ureter can cause hydronephrosis + potential kidney damage

A client is admitted with severe injuries resulting from an auto accident. The client's vital signs are BP 120/50, pulse rate 110, and respiratory rate of 28. What should be the initial nursing intervention? Administer oxygen as ordered Initiate continuous blood pressure monitoring Institute continuous cardiac monitoring Initiate the ordered intravenous therapy

Administer oxygen as ordered -early findings of shock associated w/ hypoxia + manifested by rapid HR + rapid respiration -O2 most critical initial intervention

A woman with a 28-week pregnancy is on the way to the emergency department by ambulance with a tentative diagnosis of placenta abruptio. Which action should the nurse take first when the woman arrives? Administer oxygen by mask at 100% Start a second IV with an 18 gauge cannula Check fetal heart rate every 15 minutes Insert urethral catheter with hourly urine outputs

Administer oxygen by mask at 100% -priority in this situation -it will increase the oxygen in the mother's + fetus' circulation -will minimize complications of hypoxia

A client with chronic kidney disease (CKD) is scheduled for hemodialysis at 9 am. It is now 6:30 am and the client is eating breakfast. How should the nurse help the client to prepare for hemodialysis? (Select all that apply.) Hold all oral medications Administer prescribed phosphate binder Administer prescribed vitamin D Assess patency of the access site Weigh the client Ensure the client eats a high fiber, high protein breakfast

Administer prescribed phosphate binder Administer prescribed vitamin D -to help control both serum calcium + phosphate levels Assess patency of the access site Weigh the client avoid foods high in fiber or protein - want foods that are easily digestible

The nurse is caring for a client admitted to the acute care setting with a diagnosis of Guillain-Barré. While reviewing the client's chart, which of the following orders would the nurse question? Schedule surgery for a tracheostomy Administer pyridostigmine (Mestinon) Obtain vital signs prior to plasmapheresis Physical therapy and occupational therapy consults

Administer pyridostigmine (Mestinon) anticholinesterase/cholinesterase inhibitor med for MG

A client with a documented pulmonary embolism has the following arterial blood gases (ABG): PaO2 70 mm Hg, PaCO2 30 mm Hg, pH 7.48, SaO2 87%, HCO3 22. Based on this data, what is the first nursing action? Review prior ABG data from the prior shift Have the client do slow, deep breathing Notify the health care provider of the results Administer the PRN oxygen by nasal cannula

Administer the PRN oxygen by nasal cannula

The public health nurse is reviewing data about trends in tuberculosis (TB) in the U.S. Which factor should the nurse understand about the current trends in TB? All TB clients should be counseled and tested for HIV There is no significant difference between TB rates for foreign-born and U.S.-born people The number of new cases of TB in the U.S. continues to increase each year NonHispanic Asians have the lowest TB case rate

All TB clients should be counseled and tested for HIV

The client with lymphoma is admitted for treatment of tumor lysis syndrome. Which medication will protect the kidneys by minimizing uric acid deposits? Furosemide (Lasix) Allopurinol (Aloprim, Zyloprim) Hydralazine Mannitol (Osmitrol)

Allopurinol (Aloprim, Zyloprim) -only this med reduces uric acid production

A 2 year-old child has recently been diagnosed with cystic fibrosis. A nurse is teaching the parents about home care for the child. Which piece of information is appropriate for the nurse to include? Restrict activities to inside the house Limit exposure to other children Allow the child to continue normal activities Schedule frequent rest periods

Allow the child to continue normal activities -physical activity is important in a 2 yr old who is developing autonomy -exercise tends to stimulate mucus secretion + helps develop normal breathing patterns

The nurse is reviewing age-appropriate diagnoses for older adults. Which nursing diagnosis would indicate that the client is at greatest risk for falling? Alteration in mobility related to fatigue Impaired gas exchange related to retained secretions Altered patterns of urinary elimination related to nocturia Sensory perceptual alterations related to decreased vision

Altered patterns of urinary elimination related to nocturia -nocturia especially problematic b/c many OA's fall when they rush to reach br during night -may be confused or not fully alert b/c of having been asleep

The emergency department is sending a client with a diagnosis of delirium tremens to the floor. The admitting nurse would expect which of the following findings? Single or multiple jerks caused by rapid contracting muscles with alternating relaxation An excited state accompanied by disorientation, hallucinations and tachycardia Disorganized thinking, feelings of terror and non-purposeful behavior A generalized shaking of the body accompanied by repetitive thoughts expressed verbally

An excited state accompanied by disorientation, hallucinations and tachycardia

The nurse works in the pediatric emergency department. In which situation would a child be treated by using enemas followed by an antitoxin? A toddler who has eaten an undetermined number of ibuprofen tablets A school-aged child who has swallowed a handful of iron-fortified vitamins An infant who is diagnosed with botulism A preschooler who bit into a laundry detergent pod

An infant who is diagnosed with botulism -food-borne botulism can be treated by removing w/e contaminated food is in the stomach by using enemas + administering a botulinum antitoxin

The nurse is reinforcing instructions to a 67 year-old client with type 2 diabetes. The client should be instructed to contact the outpatient clinic immediately if which of the following findings are present? Insomnia with daytime fatigue Temperature of 99.5 F (37.5 C) with painful urination Nausea with two episodes of vomiting An open, reddened wound on the heel

An open, reddened wound on the heel -w/o tx serious infection, gangrene, limb loss + sepsis may result

The nurse is providing the client who takes digoxin and furosemide with dietary instructions. The nurse should reinforce that the combination of these medications can result in which outcome? Oliguria Arrhythmias Weight gain Irritability and excitability

Arrhythmias -exaggerates metabolic effects of hypokalemia, especially alterations in cardiac rate + rhythm

The nurse is caring for an 81 year-old client with colorectal cancer. Until now, the client's pain has been managed using acetaminophen with codeine. Because of increased pain, intravenous morphine has now been ordered. What should the nurse recognize about the validity of this order? Appropriate despite the expected effect of mental confusion Inappropriate and demonstrates poor knowledge of pain control Inappropriate because of potential respiratory depression Appropriate pain management around-the-clock

Appropriate pain management around-the-clock -OA's w/ CA pain frequently undermedicated

The nurse is caring for a client who has a history of asthma and is now diagnosed with gastroesophageal reflux disease (GERD). Which of these types of medications, which are all prescribed for the client, may aggravate the GERD? Histamine blocker Anticholinergic bronchodilator Corticosteroid Anti-infective

Anticholinergic bronchodilator -will decrease gastric emptying + diminish the pressure on the lower esophageal sphincter -this will enhance gastric reflux

A client, admitted with palpitations and dyspnea, is diagnosed with atrial fibrillation (AF). Normal sinus rhythm is later restored using pharmacologic interventions. In addition to controlling cardiac rate and rhythm, the nurse understands that treatment for AF must include which of the following approaches? Anticoagulation Catheter ablation Coronary artery bypass surgery Cardioversion

Anticoagulation -significantly reduces risk of stroke + other thromboembolic events -when pt does not respond to pharmacological interventions to restore sinus rhythm, cardioversion used

A nurse notes sudden onset confusion in an 83 year-old client. Which recently ordered medication would have most likely contributed to this change? Anticoagulant Antihistamine Liquid antacid Cardiac glycoside

Antihistamine -OA's more susceptible to the s/e of anticholinergic drugs (such as antihistamines) -often cause confusion in OA

The nurse is preparing to administer albuterol inhaled to a 11 year-old with asthma. Which assessment by the nurse indicates there is a need for the health care provider to adjust the medication? Temperature of 101 F (38.3 C) Lethargy Apical pulse of 112 Lower extremity edema

Apical pulse of 112

The client is being treated for tuberculosis (TB). Which assessment would indicate that the client is having a possible adverse response to isoniazid? Severe headache Tachycardia Appearance of jaundice Decreased hearing

Appearance of jaundice -pt's being treated w/ isoniazid @ risk for developing drug-induced hepatitis

The nurse is assessing a client with portal hypertension. Which findings should the nurse expect during the assessment? Ascites Blurred vision Expiratory wheezes Dilated pupils

Ascites -portal HTN can lead to ascites from the increased portal pressure as well as a lowered colloid osmotic pressure b/c of low albumin

A client is newly admitted with acute diarrhea and is wearing an adult incontinence pad. What action should the nurse take before visitors arrive and enter the client's room? Remind all visitors to wash their hands before entering, and when exiting, the room Verify that each visitor is wearing a mask if less than 3 feet (0.9 meters) from the client Ask each visitor to dress in a gown and wear gloves before entering the client's room Call the health care provider to request an order for a private room

Ask each visitor to dress in a gown and wear gloves before entering the client's room -washing hands is very important, but not enough to protect against potential enteric pathogens

The perioperative nurse must place the anesthetized client into the lithotomy position for a cystoscopic procedure. What is the safest technique for moving the client into this position? Gently externally rotate the hips and flex the knees one at a time before placing in padded stirrup Abduct the legs and then flex the knee of one leg before placing in padded stirrup; repeat with other leg First raise one leg, flex the knee and place leg in a padded stirrup; repeat with other leg Ask for assistance to raise both legs simultaneously, then to flex both knees and place legs in padded stirrups

Ask for assistance to raise both legs simultaneously, then to flex both knees and place legs in padded stirrups -prevents excessive stretching + potential nerve damage

The nurse is planning care for a 3 month-old infant in the immediately postoperative period after the placement of a ventriculoperitoneal shunt for hydrocephalus. The nurse should take which action with anticipation of complications of the procedure? Begin formula feedings when infant is alert Pump the shunt at intervals to assess for proper function Assess for abdominal distention or taunt abdominal wall Maintain the infant in an upright position in a car seat

Assess for abdominal distention or taunt abdominal wall -b/c cerebrospinal fluid could cause peritonitis or a postop ileus as a complication of distal cath placement

A client with cirrhosis of the liver underwent a paracentesis yesterday. Today, the unlicensed assistive personnel (UAP) reports the client is lethargic and has musty-smelling breath. Which assessment should the nurse perform next? Measure the abdominal girth changes Assess for flap-like tremors of the hands Auscultate the upper abdomen for bruits Monitor the client's clotting status

Assess for flap-like tremors of the hands -subtle changes in mental status + a musty odor to the breath are findings associated w/ hepatic encephalopathy

An 88 year-old client is admitted to the telemetry unit following a minor surgical procedure. The client's history includes insulin dependent diabetes and a previous myocardial infarction. The nurse responds to the client's ECG alarm and finds the client's rhythm shows asystole and the client obtunded but responsive. Prioritize the actions of the nurse (with 1 being the top priority). Look at a different ECG lead to confirm rhythm Check a blood glucose level Assess RR + pulse Initiate emergency response system if indicated

Assess respirations and pulse Initiate emergency response system if indicated Look at a different ECG lead to confirm rhythm Check a blood glucose level

A child is injured on the school playground and appears to have a fractured leg. Which of the following is the first action a school nurse should take? Assess the child and the extent of the injury Apply cold compresses to the injured area Call for emergency transport to the hospital Immobilize the limb and joints above and below the injury

Assess the child and the extent of the injury

Postoperative orders for a client who had a mitral valve replacement include monitoring pulmonary artery pressure together with pulmonary capillary wedge pressure with a pulmonary artery catheter. What is the purpose of these actions by a nurse? Compare the right ventricular pressures Determine the changes in an acid-base balance Establish coronary artery stability Assess the left ventricular end-diastolic pressure

Assess the left ventricular end-diastolic pressure -pulmonary cap wedge pressure is reflective of LV end-diastolic pressure

A nurse is caring for a client in the coronary care unit. The display on the cardiac monitor indicates ventricular fibrillation. What should the nurse do first? Assess the level of consciousness Initiate CPR Assess the pulse Perform defibrillation

Assess the level of consciousness -always treat pt not monitor

The nurse is caring for a postoperative client who had a laparotomy. Which of these actions is the most effective nursing intervention to prevent atelectasis from developing? Assist the client to slowly deep breathe and cough Splint the incision Ambulate client within 12 hours postop Maintain adequate hydration

Assist the client to slowly deep breathe and cough -will expand lungs + stimulate surfactant production -this is the priority to prevent pulmonary complications after surgery

A client, who is nearing death, believes in Hinduism. The nurse should plan for which action? At the time of death, a priest will trickle milk or holy water into the mouth of the client The elders may be with the client during the process of dying and no last rites are given The family must be with the client during the process of dying and are responsible to wash the body at least three times after death From the moment of death, the body is never left alone until after burial, which usually occurs within 24 hours of death

At the time of death, a priest will trickle milk or holy water into the mouth of the client -mormons are w/ the pt during the dying process -washing pt after death r/t Islamic religion -body never left alone r/t Jewish groups

The client has received fentanyl, atropine and benzocaine for an endoscopic procedure. The nurse is monitoring the client and notes the pulse has increased from the preprocedure baseline. Which med could cause an increased pulse rate?

Atropine -anticholinergic -can increase HR + dilate pupils

A nurse is preparing to perform a physical examination on an 8 month-old who is sitting contentedly on the mother's lap. Which assessment should the nurse do first? Elicit the reflexes Examine the ears Measure the height and weight Auscultate the heart and lungs

Auscultate the heart and lungs

A client admits to benzodiazepine dependence for over the past several years. The client is now in an outpatient detoxification program. The nurse should understand that a priority during withdrawal from any substance is which of these actions by the client? Avoid alcohol use during this time Discontinue the drug by weaning Rise slowly from a lying to standing position Expect mild physical symptoms

Avoid alcohol use during this time -CNS depressants interact w/ alcohol

The nurse is providing information to clients taking chlorpromazine. Which teaching point should the nurse emphasize regarding this medication? Avoid tyramine-containing foods Avoid direct sunlight Avoid foods fermented with yeast Avoid canned citrus fruit drinks, especially grapefruit juice

Avoid direct sunlight -phenothiazines increase sensitivity to the sun + makes pt's especially susceptible to sunburn -may also decrease pt's ability to sweat, making them more likely to get heat stroke

The client is diagnosed with gastroesophageal reflux disease (GERD). Which recommendation made by the nurse would be most helpful? Maintain a diet of soft foods and cooked vegetables Avoid liquids unless a thickening agent is used Sit upright for at least half an hour after eating Avoid eating two hours before going to sleep

Avoid eating two hours before going to sleep -allows for stomach emptying -sitting upright for @ least 1/2 hr after eating = intervention for pt w/ swallowing difficulties

A nurse is teaching a client to select foods rich in potassium to prevent digitalis toxicity. Which choice indicates the client understands this dietary requirement and recognizes which foods are highest in potassium? Naval orange Baked potato Small banana Three apricots

Baked potato

A mother with a Roman Catholic belief system has given birth in an ambulance on the way to the hospital. The neonate is in very critical condition with little expectation of surviving the trip to the hospital. Which of these requests should the nurse in the ambulance anticipate and be prepared for? Point the neonate's face toward Mecca Baptize the neonate on the way to the hospital Place a thread around the neonate's neck or wrist Arrange for a church elder to be at the hospital for a "laying on of hands"

Baptize the neonate on the way to the hospital -laying of hands = mormon

A pregnant woman is advised to increase the intake of protein and vitamin C to meet the needs of the growing fetus. Which diet best satisfies the client's requirements? Beef, 1/2 cup lima beans, 8-ounce glass of skim milk, 3/4 cup strawberries 3 ounces chicken, 1/2 cup corn, lettuce salad, small banana 1 cup macaroni, 3/4 cup peas, glass whole milk, medium pear Scrambled egg, hash browned potatoes, large nectarine

Beef, 1/2 cup lima beans, 8-ounce glass of skim milk, 3/4 cup strawberries -beef + beans excellent source of protein -strawberries good source of vitamin C

A nurse is reinforcing teaching to a 24 year-old woman receiving acyclovir for a herpes simplex virus type 2 infection. Which of these instructions should the nurse give the client? Complete the entire course of the medication for an effective cure Stop treatment if she thinks she may be pregnant to prevent birth defects Begin treatment with acyclovir at the onset of symptoms of recurrence Continue to take prophylactic doses for at least five years after the diagnosis

Begin treatment with acyclovir at the onset of symptoms of recurrence -when the pt is aware of early s/s (pain, itching, tingling) tx is very effective -medications for herpes do not cure the dx

The nurse working in the intensive care unit (ICU) is told that a client is being newly admitted with a diagnosis of hyperglycemic hyperosmolar nonketotic state (HHNS). The nurse would expect which of the following clinical findings in this client? (Select all that apply.) Blood glucose level of at least 600 mg/dL Ketonuria History of type 1 diabetes mellitus Metabolic acidosis Severe dehydration

Blood glucose level of at least 600 mg/dL Severe dehydration -usually presents in OA's w/ type 2 diabetes -all pt's w/ HHNS require hospitalization + rapid tx to correct profound hypovolemia + hyperglycemia of this condition

The nurse is caring for a child who is diagnosed with coarctation of the aorta. Which finding would the nurse expect when assessing the child? Diminished carotid pulses Normal femoral pulses Bounding pulses in the arms Strong pedal pulses

Bounding pulses in the arms -coarctation of the aorta (narrowing or constriction of the descending aorta) -causes increased blood flow to the UE, resulting in a bonding pulse in the arms

A 4 month-old infant is being given digoxin. The client's blood pressure is 92/78 mm Hg; resting pulse is 78 BPM; respirations are 28 BPM; and the serum potassium level is 4.8 mEq/L (4.8 mmol/L). The client is irritable and has vomited twice since the morning dose of digoxin. Which finding is most indicative of digoxin toxicity? Bradycardia Vomiting Dyspnea Irritability

Bradycardia -most common sign of dig toxicity in kids

A child is sent to the school nurse by a teacher who has a written note that fifth disease is suspected. Which of the following should the nurse expect to find if the child has this condition? Koplik spots appear first followed by a rash that appears first on the face and spreads downward Discrete rose pink macules will appear first on the trunk and fade when pressure is applied Bright red cheeks, with a "slapped face" appearance Macule that rapidly progresses to papule and then vesicles

Bright red cheeks, with a "slapped face" appearance -5th dx also referred to as parvovirus infection or erythema infectiosum -slapped cheak dx

A nurse is performing an assessment on a client with pneumococcal pneumonia. Which finding should the nurse anticipate? Bronchial breath sounds in the outer lung fields Hacking, nonproductive cough Decreased tactile fremitus Hyper-resonance of areas of consolidation

Bronchial breath sounds in the outer lung fields

The client was admitted 2 days ago after a CT scan of the head revealed a basilar skull fracture (BSF). What assessment findings does the nurse anticipate with a BSF? (Select all that apply). Bruising behind the ear (Battle's sign) Unilateral redness and swelling over the mastoid bone Purulent drainage from the ear Facial numbness Hearing loss Bruising around both eyes (Raccoon eyes)

Bruising behind the ear (Battle's sign) Facial numbness Hearing loss Bruising around both eyes (Raccoon eyes)

The client is a new admission diagnosed with Alzheimer's disease (AD). The nurse reviews all drugs (including complementary and alternative treatments or therapies) routinely taken at home with a family member. Which of the following treatments would be a concern for the nurse? Ginkgo biloba Coconut oil Omega-3 fatty acids Donepezil (Aricept)

Coconut oil

The nurse is caring for a school-aged child with a diagnosis of secondary hyperparathyroidism after treatment for chronic renal disease. Which serum lab data should receive priority attention by the nurse? Glucose and potassium Osmolality and sodium Blood urea nitrogen and magnesium Calcium and phosphorus

Calcium + phosphorus -parathyroid regulates Ca + phosphorus serum levels -think of a see-saw: Ca 1st in alphabet + thus Ca follows direction of abnormality (hyper/hypo function)

A 36 year-old female client has a hemoglobin level of 14 g/dL and a hematocrit of 42%, 24 hours after a dilation and curettage (D&C). Which of the following findings should the nurse expect when assessing the client? Complaints of fatigue with ambulation Capillary refill of less than three seconds Pale mucous membranes Respirations 36 breaths per minute

Capillary refill of less than three seconds

The health care provider orders the antidepressant trazodone ER 150 mg at bedtime. Which common side effect of this drug should the client understand? Reduces arthritic pain Causes drowsiness Relieves nasal stuffiness Decreases acne breakouts

Causes drowsiness -has sedative effects

A 6 month-old infant who is being treated for developmental dysplasia of the hip has been placed in a hip spica cast. What should the nurse be sure to teach the parents about caring for their child at home? Check every few hours for the next day or two for swelling in the baby's feet Gently rub the skin with a cotton swab to relieve itching Turn the baby with the abduction stabilizer bar every two hours Place the favorite books and push-pull toys in the crib

Check every few hours for the next day or two for swelling in the baby's feet -must be checked for circulatory impairment

pon entering an adult client's room, the client is found to be unresponsive. After calling for help, what is the next action that should be taken by the nurse? Check for a carotid pulse Give two rescue breaths Deliver five abdominal thrusts Maintain an open airway

Check for a carotid pulse

A client is diagnosed with amyotrophic lateral sclerosis (ALS). Which nursing action will help prevent the complications of atelectasis and pneumonia in this client? Chest physiotherapy twice a day Use of the incentive spirometer every two hours while awake Repositioning every two hours around the clock Active and passive range of motion exercises twice a day

Chest physiotherapy twice a day -repositioning pt more r/t circulation issues

The client with cancer is being treated with a biological response modifier. Which of the following side effects does the nurse anticipate with biologic therapy? Photophobia and sun sensitivity Constipation Hematuria Chills and fever

Chills and fever -biological response modifier CA therapy agents are drugs that stimulate body's own defense mechanisms to fight CA cells -flu like findings (chills, fever) common s/e of this type of therapy

A nurse is teaching adolescents about sexually transmitted diseases. What should the nurse emphasize is the most common infection? Gonorrhea Human immunodeficiency virus (HIV) Herpes Chlamydia

Chlamydia

A client visits a community clinic for the treatment of recurrent pelvic inflammatory disease (PID). The nurse should plan to teach this client with the knowledge that this condition most frequently follows which type of infection? Chlamydia Syphillis Herpes simplex 2 Trichomoniasis

Chlamydia -chlamydia + gonorrhea are the most frequent cause of PID

The 72 year-old client has an estimated blood loss of 600 mL during a gastric resection. The surgeon orders two units of packed cells (PC) to be administered in the post anesthesia care unit. During the administration of the second unit of PC, the nurse notes the following findings: hypertension, a bounding pulse, and increasing dyspnea. What is the probable cause of these findings? Bacterial transfusion reaction Circulatory overload Transfusion-associated graft-versus-host disease Hemolytic transfusion reaction

Circulatory overload -OA's @ risk for circulatory overload, especially when solutions are administered rapidly -HTN w/ a bounding pulse + dyspnea = key signs of fluid overload

The nurse is providing postoperative care for a client who has undergone a laparoscopic cholecystectomy. Which assessment finding should be reported immediately to the health care provider? Client reports severe right upper quadrant tenderness Client is drowsy Absence of bowel sounds Client reports shoulder pain

Client reports severe right upper quadrant tenderness -could be from a retained gallstone or bile duct injury -sever pain in RUQ is a med emergency after lap chole -absence of bowel sounds immediately after surgery not a cause for alarm

The nurse manager is conducting a staff development program about improving client care outcomes. Which of the following processes will best contribute to improved outcomes? Clients with peripheral edema will have blood drawn for a daily chemistry panel Clients with a history of pressure ulcers will have an air mattress in place within 6 hours of admission Clients with a diagnosis of heart failure will have an indwelling urinary catheter inserted Clients will be screened upon admission for methicillin-resistant staphylococcus aureus (MRSA)

Clients with a history of pressure ulcers will have an air mattress in place within 6 hours of admission -specific example of quality nursing + preventative care -screening pt's for MRSA or blood draws are not interventions

Due to a recent outbreak in the community, the nurse is speaking to a group of parents and elementary school teachers about rheumatic fever. Which information is most important for the nurse to emphasize? Children may remain strep carriers for years Most play activities will be restricted indefinitely Clumsiness and behavior changes should be reported Home schooling is preferred to classroom instruction

Clumsiness and behavior changes should be reported -sydenham chorea is a major sign of acute rheumatic fever (jerky, uncontrollable, purposeless mvmts that look like twitches, loss of fine motor control)

The nurse is caring for a client who is experiencing a hypertensive crisis. The priority assessment in the first hour of care after admission to the critical care unit should focus on which factor? Lung sounds Heart rate Pedal pulses Cognitive function

Cognitive function -the organ most susceptible to damage in HTN crisis is the brain d/t rupture of cerebral bv's

The nurse is reviewing the medical records for a newborn and sees that the first APGAR score was an 8 and the second score was a 9. Which category of the APGAR test is most likely the reason for the improved score? Heart rate Cry Muscle tone Color

Color -most common APGAR score deduction

A child is to have chest physiotherapy (CPT) by the nurse. Which nursing action is appropriate? Place the child in a prone position for the therapy Teach the child not to cough during the treatment Confine the percussion to the rib cage area Schedule the therapy 30 minutes after meals

Confine the percussion to the rib cage area -1 hr prior or 2 hrs after meals

A client with a history of asthma and kidney stones is admitted with a diagnosis of recurrent renal calculi. The client experiences shortness of breath following a lithotripsy. The nurse auscultates the client's lungs and finds decreased air movement but no wheezing. The arterial blood gas (ABG) results are pH 7.31, PaO2 53 mm Hg, PaCO2 50 mm Hg, and O2 sat 82%. Which of the following actions are appropriate for the nurse to take? (Select all that apply.) Contact the health care provider Start high flow oxygen via face mask Increase IV fluids Start oxygen via nasal cannula Administer a short-acting bronchodilator via nebulizer Prepare for possible intubation Call respiratory therapy

Contact the health care provider Start high flow oxygen via face mask Administer a short-acting bronchodilator via nebulizer Prepare for possible intubation Call respiratory therapy

The nurse reviews a new order: infuse 1000 mL D5W with 100 mEq KCl intravenously at a rate of 50 mL/hour. What action should the nurse take? Contact the prescriber and point out a concern about the amount of KCl ordered Verify the information in the order and prepare to administer the IV solution Ask another nurse to observe adding 100 mEq KCl to the IV solution Ask another nurse for a second opinion about the order before checking the client's labs

Contact the prescriber and point out a concern about the amount of KCl ordered -no indication if IV solution is for peripheral or central admin -max concentration of KCl via peripheral line is ~ 10 mEq/100 mL + 10 mL/hr

A client who is terminally ill has been receiving high doses of an opioid analgesic for the past month. As death approaches and the client becomes unresponsive to verbal stimuli. What orders should the nurse expect from the health care provider? Discontinue the analgesic Continue the same analgesic dosage Prescribe a less potent drug Decrease the analgesic dosage by half

Continue the same analgesic dosage -pt's who are actively dying + have been experiencing chronic pain, will probably continue to experience pain even though they cannot communicate this

A client has a chest tube inserted immediately after surgery for a left lower lobectomy. During the repositioning of the client during the first postop check, the nurse notices 75 mL of a dark, red fluid flowing into the collection chamber of the chest drain system. What is the appropriate nursing action? Turn the client back to the original position Continue to monitor the rate of drainage Check to see if the client has a type and cross match Call the surgeon immediately

Continue to monitor the rate of drainage -not unusual for blood to collect in the chest + be released into the chest drain when the pt changes position this soon after surgery -dark color of the blood indicates it is not active bleeding inside the chest

A mother wants to switch her 9 month-old infant from an iron-fortified formula to whole milk. Upon further assessment, a nurse finds that the baby eats table foods well, but drinks less milk than before. What information is the priority for the nurse discuss with the mother? Change the baby to whole milk Continue with the present infant formula Offer fruit juice in a sippy cup occasionally Continue introducing new food

Continue with the present infant formula -even though baby is eating table food + may be drinking less formula, she still needs to get most of her nutrition from formula (or breast milk) -don't switch milk until 12 months -babies can't digest cows milk

The parent of a 2 year-old reports the child has experienced mild diarrhea for the past two days. Which statement by the nurse provides the best nutritional information for the child? Offer bananas, apples, rice and toast as tolerated Continue with the regular diet and include oral rehydration fluids Clear liquids and gelatin for 24 hours NPO for 24 hours, then rehydrate with milk and water

Continue with the regular diet and include oral rehydration fluids -current recommendations for mild-mod diarrhea are to maintain a normal diet w/ fluids to rehydrate -if diarrhea was severe then the BRAT (bananas, apples, rice + toast) diet would be appropriate

A nurse is caring for a client who is receiving procainamide intravenously. It is important for the nurse to monitor which of these parameters? Continuous ECG readings Neurological signs Serum potassium levels Hourly urinary output

Continuous ECG readings -procainamide used to suppress cardiac arrhythmias -when administered, it must be accompanied by continuous cardiac monitoring

The nurse reviews an order to administer Rh (D) immune globulin to an Rh negative woman after the birth of her Rh positive newborn. Which assessment is a priority before the nurse gives the injection? Coombs test results Previous RhoGAM history Gravida and parity Newborn's blood type

Coombs test results -negative test confirms that antibodies have not been formed in the new mother

The nurse is caring for a child diagnosed with Kawasaki disease (mucocutaneous lymph node syndrome or infantile polyarteritis). The nursing care plan should be based on the knowledge that this child is at risk for developing what complication? Chronic vessel plaque formation Coronary artery aneurysm Occlusions at the vessel bifurcations Pulmonary embolism

Coronary artery aneurysm -Kawasaki dx can cause inflammation of arteries (especially coronary arteries) -can lead to aneurysms

During assessment of a postpartum client, the nurse palpates a firm fundus and observes a constant trickle of bright red blood from the vagina. What is the most likely cause of these findings? Genital lacerations Retained placenta Uterine atony Clotting disorder

Genital lacerations -continuous trickling of blood in the absence of a boggy fundus undetected genital tract lacerations

A nurse is teaching a child and family members about the medication phenytoin prescribed for seizure control. Which side effect is most likely to occur? Drowsiness Vomiting Vertigo Gingival hyperplasia

Gingival hyperplasia (overgrowth of the gums)

A nurse is assigned to care for a client who had a myocardial infarction (MI) two days ago. The client has many questions about this condition. What area is a priority for the nurse to discuss with the client at this time? The overview cardiac rehabilitation Daily physical and mental needs and concerns Activity and rest guidelines Medication and diet guideline

Daily physical and mental needs and concerns -@ 2 days after MI, pt's education should be focused on the immediate needs + concerns for the day

A couple attempting to conceive asks the nurse when ovulation occurs. The woman reports a regular 32-day cycle. Which response by the nurse is correct? Days 14-16 Days 7-10 Days 11-13 Days 17-19

Days 17-19 -ovulation occurs 14 days prior to menses -subtract 32 from 14

The parents of a 6 year-old child who normally enjoys school tells a nurse that the child has not been doing well since a grandmother died two months ago. Which statement most accurately describes thoughts on death and dying at this age? Death is personified as the bogeyman or devil The child is worried that the child, too, might die Death is perceived as being irreversible The child feels guilty for the grandmother's death

Death is personified as the bogeyman or devil -personification of death is typical of this developmental level -this age is @ the end of the prek period where magical thinking for the animation of inanimate objects is present

There is an order to administer atropine to a client preoperatively. The nurse understands that which effect is the intended purpose for giving this medication preoperatively? Elevate blood pressure Decrease secretions Enhance sedation Reduce heart rate

Decrease secretions -atropine is a common anesthesia adjunct -decreases amt of secretions - which decreases risk of aspiration during operative procedure

A client has been taking furosemide for the past week. The nurse recognizes that which finding may indicate the client is having a negative side effect from the medication? Gastric irritability Edema of the ankles Weight gain of five pounds Decreased appetite

Decreased appetite -findings of hypokalemia include anorexia, fatigue, nausea, decreased GI motility, muscle weakness

The nurse is caring for a client who is unconscious and receiving gastric tube feedings. Which assessment finding requires an immediate action from the nurse? Urine output of 250 mL in the past eight hours Decreased breath sounds in the right lower lobe Formula residual volume 100 mL Decreased bowel sounds in all quadrants

Decreased breath sounds in the right lower lobe -most common problem associated w/ enteral feedings is aspiration w/ resulting atelectasis + pneumonia -maintain pts @ minimum of 30 degrees HOB during feedings + 2 hrs after

The nurse suspects that the client is in cardiogenic shock. Which of the following findings supports this information? Decreased or muffled heart sounds Bounding pulses Bradycardia Increased cardiac output

Decreased or muffled heart sounds -cardiogenic shock involves decreased CO + evidence of tissue hypoxia -hypotension, rapid + faint pulses, distant sounding heart sounds

A client is brought to the emergency department with a blood sugar of 52 mg/dL. The client is weak and diaphoretic but awake, and the client's blood sugar does not rise above 70 mg/dL after drinking one 4-ounce glass of orange juice. Which of the following actions should be taken? (Select all that apply.) Determine blood sugar management medications Recheck blood sugar in 15 minutes Instruct the client to not take more insulin today Offer 8-ounce glass of milk Offer a 12-ounce can of cola with added sugar

Determine blood sugar management medications Recheck blood sugar in 15 minutes Offer 8-ounce glass of milk

The nurse is caring for a client newly diagnosed with atrial fibrillation. The atrial heart rate is 250 and the ventricular rate is controlled at 75. Which finding is a cause for the most concern? Diminished bowel sounds Difficulty speaking Tachypnea with movement Loss of appetite

Difficulty speaking -may indicate pt has suffered a stroke -atrial rate of 250 is normal for afib + is of no concern -ventricular rate of 75 indicates that the cardiac rate is well controlled

A nurse is reviewing the medication administration record (MAR) for a 72 year-old client scheduled to have a cardioversion. Prior to the procedure, the nurse should notify the health care provider if the client received which medication during the preceding 24 hours? Nitroglycerin ointment Diltiazem (Cardizem) Digoxin (Lanoxin) Metoprolol (Toprol XL)

Digoxin (Lanoxin) -increases ventricular irritability + increases risk of vfib during + after cardioversion

On the second postoperative day, a 79 year-old female (who was previously cognitively intact) becomes agitated when she begins having auditory and visual hallucinations. The client later demands to leave the hospital. What action does the nurse take next? (Select all that apply.) Discuss the situation with the durable power of attorney for healthcare (DPOAHC) Correct response Describe the risks and benefits of leaving the hospital to the client Inform the client that insurance will deny payment for leaving against medical advice (AMA) Contact the primary physician to evaluate the client Review the release from liability form with the client and witness her signature

Discuss the situation with the durable power of attorney for healthcare (DPOAHC) Correct response Describe the risks and benefits of leaving the hospital to the client -pt has right to leave AMA Contact the primary physician to evaluate the client

The nurse notes that a prescription for captopril was changed to losartan, even though the captopril provided effective blood pressure control. What is the most likely reason for discontinuing the captopril? Dry cough Sexual dysfunction Rash and itching Blurred vision

Dry cough -rash, itching + blurred vision common s/e

The nurse is assisting with the delivery of a newborn infant. What is the priority nursing intervention for a normal newborn immediately after delivery? Dry off infant with a warm blanket or towel Assign the one-minute APGAR score Obtain vital signs Apply identification bracelets

Dry off infant with a warm blanket or towel -maintaining infant's temp -warming + drying are the priority

A female client diagnosed with genital herpes simplex virus 2 (HSV-2) reports having dysuria, dyspareunia, leukorrhea and lesions on the labia and perianal skin. Which intervention will provide symptomatic relief? (Select all the apply.) Soak in a tub of hot water Increase fluid intake Dry the genital area with a blow dryer on the cool setting Echinacea juice extract capsules twice daily Local application of ice packs Over-the-counter medications such as ibuprofen

Dry the genital area with a blow dryer on the cool setting -can help relieve s/s Local application of ice packs Over-the-counter medications such as ibuprofen -can help w/ local tenderness

Recurrent weeping and oozing skin papules and vesicles that later become crusted and scaly

Eczema

The client is diagnosed with superficial thrombophlebitis of the left leg. Which nursing intervention should be given the highest priority? Apply elastic support stockings Elevate the affected leg Maintain complete bed rest Apply cool compresses

Elevate the affected leg -b/c dangling extremity will increase swelling + pain

The nurse compares the third postoperative assessment findings to the first two postoperative assessments. What action should the nurse take to provide optimal care for this client? (Select all that apply.) Blood Pressure Pulse Respiratory Rate Oxygen Saturation 1st Postop Assessment 110/80 mm Hg 80 10 98% 2nd Postop Assessment 100/72 mm Hg 88 16 97% 3rd Postop Assessment 92/64 mm Hg 106 24 95% Elevate the client's lower extremities Administer pain medication Administer an intravenous fluid bolus Assist the client to use the incentive spirometer Move the bed into Trendelenburg position Inspect the surgical incision site

Elevate the client's lower extremities Administer an intravenous fluid bolus -to increase volume Inspect the surgical incision site

A nurse is caring for a client with schizophrenia who has been treated with quetiapine for one month. Today the client is increasingly agitated and reports having muscle stiffness. Which of these additional findings should be reported to the health care provider? Elevated temperature and sweating Decreased pulse and blood pressure Mental confusion and general weakness Muscle spasms and seizures

Elevated temperature and sweating -NMS

The clinic nurse is caring for a 15 month-old child with a first episode of otitis media. Which intervention should the nurse include in the instructions to the child's parents? Emphasize the importance of a return visit after completion of antibiotics Explain that the child should complete the full five days of antibiotics Provide them with handout describing care of myringotomy tubes Describe the tympanocentesis to detect persistent infections

Emphasize the importance of a return visit after completion of antibiotics -should be examined again after completion of full course of antibiotics to assess for persistent infection or middle ear effusion -usual tx for otitis media is oral antibiotics for 10-14 days

A nurse is teaching parents about accidental poisoning in children. Which action should the nurse emphasize that the parents initially take if there is a suspected poisoning? Keep the child as quiet as possible if a toxic substance was inhaled Do not induce vomiting if the poison is a hydrocarbon Call the Poison Control Center once the situation is identified Empty the child's mouth in any case of a possible poisoning

Empty the child's mouth in any case of a possible poisoning -prevents further ingestion + should be done first to limit damage from substance

A nurse is caring for a 13 year-old after a spinal fusion to treat scoliosis. Which nursing intervention is appropriate in the immediate postoperative period? (Select all that apply.) Perform neurovascular checks every 8 hours Encourage passive leg and ankle exercises Position the client flat in bed and logroll every 2 to 4 hours Assist the client to stand and walk to the bathroom as needed Maintain bedrest with the head of the bed elevated at least 30 degrees Encourage use of patient-controlled analgesia

Encourage passive leg and ankle exercises Position the client flat in bed and logroll every 2 to 4 hours Encourage use of patient-controlled analgesia`

The nurse is caring for a client with acute pancreatitis. After pain management, which intervention should be included in the plan of care? Institute seizure precautions Provide a diet high in protein Encourage the client to cough and deep breathe every two hours Place the client in contact isolation

Encourage the client to cough and deep breathe every two hours -respiratory infections are a common complication of pancreatitis b/c fluid in the retroperitoneum can push up against the diaphragm, causing shallow respirations -C + DB will diminish the occurrence of this complication

The oncology client is using patient controlled analgesia (PCA) with morphine for pain control. The client reports having pain and states it is a 7 (on a scale of 0 to 10). Indicate the correct sequence of nursing interventions by dragging and dropping the sentences in the correct order. Assess LOC + respiratory status Consult w/ HCP Check chart for orders for treating breakthrough pain Confirm that there is power to the pump + tubing is patent Ensure pt is using PCA equipment properly

Ensure pt is using PCA equipment properly Confirm that there is power to the pump + tubing is patent Assess LOC + respiratory status Check chart for orders for treating breakthrough pain Consult w/ HCP

The nurse is preparing a client diagnosed with deep vein thrombosis (DVT) for a venous doppler evaluation. Which of these actions should be necessary to prepare the client for this test? Ensure the client is wearing a hospital gown prior to the test Determine if the client has any allergies to the contrast material Ask client not to eat or drink anything after midnight Administer a sedating medication prior to the test

Ensure the client is wearing a hospital gown prior to the test -pt may drink/eat before test -noninvasive test + does not require sedation

A client initially experiences a large local reaction with swelling of the entire leg after being stung by a bee. A concerned family member drives the client to the emergency department. The client is now having difficulty breathing and has swelling of the tongue. Which of the following medications should be administered first? Diphenhydramine (Benadryl) subQ Methylprednisolone (Solu-Medrol) IV Epinephrine (Adrenaline) IV Albuterol (Proventil) inhaler

Epinephrine (Adrenaline) IV -albuterol for bronchospasm w/o obstruction

The client is admitted with a diagnosis of ulcerative colitis. Which laboratory values should the nurse be sure to check? (Select all that apply.) Blood urea nitrogen (BUN) Erythrocyte sedimentation rate (ESR) Albumin T3 and T4 count Hematocrit and hemoglobin White blood cell count (WBC)

Erythrocyte sedimentation rate (ESR) Albumin Hematocrit and hemoglobin White blood cell count (WBC)

A 6 year-old child with moderate edema and severe hypertension associated with acute glomerulonephritis (AGN) is admitted to the hospital. Which intervention would be the priority for the nurse? Establish seizure precautions Relieve boredom through physical activity Administer prescribed antibiotics Encourage the child to eat protein-rich foods

Establish seizure precautions -severity of acute phase of AGN variable + unpredictable -child w/ edema, severe HTN + gross hematuria may be subject to complications such as the dev of HTN encephalopathy

A nurse walks into a client's room and finds the client lying still and silent on the floor. What should the nurse do next? Assess the client's airway Incorrect Call for help Establish that the client is unresponsive Determine if anyone saw the client fall

Establish that the client is unresponsive -1st step in CPR -nurse would then call for help -check pt's pulse

The nurse is assessing a client diagnosed with chronic obstructive pulmonary disease (COPD). The client is on oxygen for low PaO2 levels. Which assessment is a nursing priority to evaluate the outcome of the therapy? Evaluate oxygen saturation (SaO2) levels frequently Assess lung sounds Observe for skin color changes Assess coughing frequency and sputum characteristics

Evaluate oxygen saturation (SaO2) levels frequently -best method to eval a pt's oxygenation is to evaluate SaO2 -most precise

During examination of the mouth of a 3 year-old child, the nurse discovers that the teeth have chalky white-to-yellowish staining with pitting of the enamel. Which of these conditions would most likely explain these findings? Oral iron therapy Ingestion of tetracycline Poor dental hygiene Excessive fluoride intake

Excessive fluoride intake

A nurse is preparing a 5 year-old for a scheduled tonsillectomy and adenoidectomy when the parents express anxiety and concern about the child's reaction to impending surgery. Which nursing intervention should best prepare the child? Arrange a tour of the operating and recovery rooms Encourage the child to bring a favorite toy to the hospital Introduce the child to all staff the day before surgery Explain the surgery one week prior to the procedure

Explain the surgery one week prior to the procedure -5 yo can understand the surgery + should be prepared well before the procedure

The nurse is teaching a 10 year-old child prior to heart surgery. Which form of explanation meets the developmental needs of this age child? Provide the child with a booklet to read about the surgery Explain the surgery using a model of the heart Provide a verbal explanation just prior to the surgery Introduce the child to another child who had heart surgery three days ago

Explain the surgery using a model of the heart -according to piaget the school-age child is in the concrete operations stage of cognitive development -use of something concrete like a model will help child understand explanation of heart surgery

The nurse is suctioning a client's tracheostomy. During this procedure, the nurse should instill saline for what purpose? Facilitate the removal of mucus plugs Decrease the client's discomfort Reduce the viscosity of secretions Prevent client aspiration from secretions

Facilitate the removal of mucus plugs -to thin + loosen the plug or viscous secretions

The nurse auscultates bibasilar inspiratory crackles in a newly admitted 68 year-old client with systolic heart failure and an ejection fraction of 30%. Which other finding is most common with this diagnosis? Fatigue Chest pain Peripheral edema Nail clubbing

Fatigue -common w/ LV heart failure d/t fluid backing up into lungs + pulmonary congestion -nail clubbing usually associated w/ disorders of the lung

An 82 year-old client reports being unable to completely empty the bladder and feeling bloated and uncomfortable. What additional finding does the nurse expect and need to report to the health care provider right away? Inadequate fiber in the diet Acid reflux and indigestion Excess abdominal fat Fecal impaction

Fecal impaction -combo of a full bladder and constipation will cause abd bloating + discomfort

A parent has numerous questions regarding normal growth and development of a 10 month-old infant. Which growth or development parameter should be of the most concern to a nurse? Head circumference is about the same as chest size Able to stand up briefly in play pen Cries when the parents leave the room Fifty percent increase in birth weight

Fifty percent increase in birth weight -hc + cc size about the same b/w 6 m-2yrs

A client, who had his entire stomach surgically removed six months ago, is now readmitted. Which of the following assessment findings would indicate that the client is experiencing complications associated with this surgery? Poor wound healing Tendency to bruise easily Decreased night vision Findings consistent with fatigue

Findings consistent with fatigue -when pt's have stomach surgically removed, they no longer have the stomach's production of intrinsic factor, leading to poor vitamin B12 absorption -results in anemia w/ s/s of fatigue d/t decreased # RBCs to carry O2 to bdy

The nurse is preparing to administer a feeding through a percutaneous endoscopic gastrostomy (PEG) tube. What nursing action is needed before starting the infusion? (Select all that apply.) Flush the tube with 30 mL of warm water Keep the feeding product refrigerated until ready to use Palpate the abdomen Milk or massage the tube Elevate the head of the bed 30-45 degrees Verify the length and placement of the tube

Flush the tube with 30 mL of warm water -not hot + not cold water Elevate the head of the bed 30-45 degrees Verify the length and placement of the tube -feeding products should be brought to room temp before infusion to prevent GI discomfort

The client has an order for benztropine. After assessing the client, which condition would contraindicate the use of this medication? Parkinson's disease Neuromalignant syndrome Glaucoma Acute extrapyramidal syndrome

Glaucoma -anticholinergic med used to treat extrapyramidal disorders caused by antipsychotic meds or PD

A nurse should question the use of atropine as a treatment for symptomatic bradycardia in which of these conditions? Glaucoma Right-sided heart failure Increased intracranial pressure Urinary incontinence

Glaucoma -atropine can cause pupillary dilation w/ an increase in aqueous humor -this leads to an increase in optic pressure

A woman in early labor puts her call light on and tells the nurse "I think my water bag just broke and I feel like something came out with the water." A visual exam by the nurse reveals a prolapsed umbilical cord. List in order of priority the actions the nurse should perform in this obstetrical emergency. Call for assistance, asking that the health care provider is notified Place the client in a knee-chest position on the bed Glove and place two fingers into the cervical opening, beside the umbilical cord, to relieve pressure Administer oxygen to the mother via mask at 10 L/min

Glove and place two fingers into the cervical opening, beside the umbilical cord, to relieve pressure Call for assistance, asking that the health care provider is notified Place the client in a knee-chest position on the bed Administer oxygen to the mother via mask at 10 L/min

Anxiety, exophthalmos, heat intolerance, restlessness, weakness

Graves' dx -from hyperthyroidism

The nurse is preparing a client and her healthy newborn for discharge. The nurse provides information about hormonal effects in newborns and tells the client to expect which of the following conditions in her baby? Edema of the scrotum Lanugo Mongolian spots Gynecomastia

Gynecomastia -exposure to maternal hormones in utero may cause temp conditions in newborn -about 3 days after birth, both newborn boys + girls may experience swelling of breasts as result of withdrawal of maternal estrogen -should go away by 2nd wk

The nurse in the physician's office is assessing a geriatric client who began taking omeprazole a month ago. Which finding indicates the drug has had the desired effect? Feelings of depression are not as severe Heartburn discomfort is lessened Chronic pain level is markedly decreased Blood pressure readings are lower

Heartburn discomfort is lessened -ome"prazole" = PPI -used to decrease stomach acid + relieve s/s of GERD

A nurse is assessing a healthy child at the two-year check up. Which finding should the nurse report immediately to the health care provider? Height and weight percentiles vary widely Growth pattern appears to have slowed Recumbent and standing height are different Short-term weight changes are uneven

Height and weight percentiles vary widely -ht + wt should be close in percentiles @ this age -wide difference may indicate a problem

The nurse is caring for a client who is involved in an abusive relationship. The nurse understands that during the "tension building" phase, the battered client may experience which of these feelings? Anger Optimism Calm Helplessness

Helplessness -in tension building phase the abuser finds more things to criticize + becomes more cruel -victims may become more compliant or withdraw; they cannot allow themselves to become angry or fight back

The nurse is caring for a client who received tenecteplase to open an occluded coronary artery following an acute myocardial infarction. Which finding should cause the greatest concern for the nurse? Red-colored tint of the urine Pink frothy sputum Serosanguineous drainage from gums Hematemesis

Hematemesis -hemorrhage is the most common risk factor associated w/ any thrombolytic

A nurse has administered several blood transfusions over three days to a 12 year-old client with thalassemia. What lab value should the nurse monitor during this therapy? Hemoglobin Platelet count Red blood cell indices Reticulocyte count

Hemoglobin -transfusions help maintain Hgb @ a high enough concentration to provide O2 to the body + prevent growth abnormalities + organ damage thalassemia = blood disorder in which body makes an abn form of Hgb

A pregnant client, at 34-weeks gestation, is diagnosed with a pulmonary embolism (PE). Which of these medications does the nurse anticipate the health care provider will initially order? Warfarin (Coumadin) therapy every other day to maintain a PT at 1.5 to 2 times the control value Subcutaneous heparin 5000 units twice a day Heparin infusion to maintain the aPTT at 1.5 to 2 times the control value Low dose aspirin therapy

Heparin infusion to maintain the aPTT at 1.5 to 2 times the control value

A nurse is assessing a 4 month-old infant. Which motor skill should the nurse anticipate finding? Hold a rattle Drink from a cup Bang two blocks Wave "bye-bye"

Hold a rattle -can grasp a toy 4-6 months

The client is a 16 year-old with full-thickness burns involving 20% total body surface area. After the initial 24 hours of treatment to replace fluids, which factor is used to determine if the client's fluid needs are being met? Hourly urine output Daily weight measurements Parkland formula for fluids Daily hematocrit results

Hourly urine output -daily wt r/t caloric intake

A client has a serum glucose of 385 mg/dL (21.4 mmol/L). Which of these verbal orders would be a priority for the nurse to question and call back the health care provider for a revision? Document peripheral glucose sticks every four hours Repeat glycosolated hemoglobin in 24 hours Humulin N 20 units IV push over 10 minutes IV fluids of 0.9% normal saline at 125 mL per hour

Humulin N 20 units IV push over 10 minutes -short acting insulin (regular insulin) is the only insulin that can be given IV

A nurse is caring for a 2 year-old child who is being treated for lead poisoning by chelation therapy. The nurse should be alert for which side effect of chelation therapy? Hypocalcemia Neurotoxicity Hepatomegaly Ototoxicity

Hypocalcemia -since chelation therapy removes minerals from the body, there is a risk of developing low calcium levels + bone damage

Dry, scaly skin; muscle cramps; tingling of the lips, fingers and toes

Hypoparathyroidism -PTH helps control serum levels of calcium, phosphorus + vitamin D

Abdominal pain; amenorrhea; decreased libido; osteoporosis; sensitivity to cold

Hypopituitarism

A group of nurses on a unit are discussing stoma care for clients who have had a stoma made for fecal diversion. Which stomal diversion poses the highest risk for skin breakdown? Sigmoid colostomy Ileostomy Ileal conduit Transverse colostomy

Ileostomy -continuous, liquid nature -high pH, alkaline output contains gastric + enzymatic agents that when present on skin can denude skin in a few hrs

The nurse is caring for a client diagnosed with a venous stasis ulcer on one leg. Which nursing intervention would be most effective to promote healing? Improve the client's nutritional status Begin proteolytic debridement within 24 hours Apply dressings with the use of sterile technique Initiate whirlpool bath therapy

Improve the client's nutritional status -healing can only be accomplished w/ proper nutrition -w/o proper nutrition, the other interventions would be of little help

A nurse prepares to administer eye drops to a 6 year-old child. Which of these descriptions describes the correct method for the instillation of eye drops? Directly on the anterior surface of the eyeball In the conjunctival sac as the lower lid is pulled down Under the upper lid as it is pulled upward In the corner where the lids meet

In the conjunctival sac as the lower lid is pulled down

The nurse is working to establish a therapeutic relationship with a client. Which of these approaches would be most damaging to the client's self-esteem? Fear Indifference Anger Disapproval

Indifference

An obese client tells the nurse: "I just started a diet and I am eating no more than 800 calories a day." What information is most important for the nurse to know in order to therapeutically respond to this statement? Individuals following a very low-calorie diet need professional monitoring This diet is classified as low calorie and adequate if balanced with 1 meat, 1 fruit, and 2 fat exchanges Very low-calorie diets often have severe and irreversible side effects A very low-calorie diet is never a successful weight loss program and should be discouraged

Individuals following a very low-calorie diet need professional monitoring -A very low-cal diet (VLCD) is a short term wt loss method for obese people -can result in a loss of ~3-5 lbs/wk

A nurse is taking a health history from parents of a child admitted with possible Reye's syndrome. Which recent illness would the nurse recognize as increasing the risk of developing Reye's syndrome? Hepatitis Rubeola Meningitis Influenza

Influenza

A child is admitted to the pediatric unit with a diagnosis of suspected meningococcal meningitis. Which admission orders should the nurse implement first? Institute seizure precautions Initiate droplet precautions Monitor neurologic status every hour Administer cefotaxime (Claforan) IV 50 mg/kg/day divided every six hours

Initiate droplet precautions

A nurse is caring for a client with active tuberculosis who has a history of noncompliance. Which of these actions by the nurse would represent appropriate care for this client? Ask the health care provider to change the regimen to fewer medications Schedule weekly clinic visits for the client Instruct the client to wear a high efficiency particulate air mask in public places

Instruct the client to wear a high efficiency particulate air mask in public places -direct observed therapy is a recognized method for ensuring pt's compliance to drug regimens

The visiting nurse makes a postpartum visit to a married female client and her husband. Upon arrival, the nurse observes that the client has a black eye and numerous bruises on her arms and legs. What should be the initial nursing intervention? Call the police to report indications of domestic violence Leave the home because of the unsafe environment Interview the client in a private place in the home to determine the origin of the injuries Confront the husband about the condition of his wife

Interview the client in a private place in the home to determine the origin of the injuries -separate suspected abused person from partner until any battering has been ruled out by convo in private location in the house -no info given of the situation that would warrant to leave or to call police

A 48-year old female with metastatic breast cancer is scheduled to receive her first dose of a trastuzumab (Herceptin). Which of the following results would prompt the nurse to hold the prescribed treatment and discuss the assessment with the ordering health care provider? Blood glucose 130 mg/dL Positive human epidermal growth factor receptor 2 (HER2) Positive lymph node involvement Irregular apical pulse

Irregular apical pulse -Trastuzumab is a monoclonal antibody used as antiCA therapy for women w/ HER2-(+) breast CA -main concern is cardiotoxicity

The nurse is teaching the client with chronic renal failure (CRF) about medications. The client questions the purpose of taking aluminum hydroxide. What is the best explanation for the nurse to give the client about the therapeutic effects of this medication? It will reduce serum calcium Amphojel increases urine output The drug is taken to control gastric acid secretion It decreases serum phosphate

It decreases serum phosphate -aluminum binds phosphates that tend to accumulate in the pt w/ chronic renal failure d/t decreased filtration capacity of kidney

The client with coronary artery disease has a prescription for nitroglycerin transdermal patches. What is the best reason the client should not wear a patch for more than 12 to 14 hours each day? It will cause profound hypotensive effects it will irritate the skin It may no longer work as well It can cause severe headaches

It may no longer work as well -nitro patches may not work as intended when they are used continuously -to prevent tolerance pt's should apply a patch 1x/day + remove it after 12-14 hrs

The nurse manager is preparing for an inservice about efficiency and cost-effective care. Identify a strategy the nurse manager should include in the presentation that's aimed at increasing efficiency and reducing costs on the nursing unit. Increase the number of unlicensed staff providing care Limit dressings packages and other supplies on the nursing unit Eliminate reimbursement for continuing education seminars Label the cost of each item in the supply closet

Label the cost of each item in the supply closet -when the cost of each item is displayed, staff becomes more aware of costs + may use supplies more wisely -eliminating staff education reimbursement will not increase efficiency + may even decrease it

A 30 month-old child is admitted to the hospital unit. Which of these toys would be appropriate for the nurse to select from the toy room for this child? Large wooden puzzle Blunt scissors and paper Beach ball Cartoon stickers

Large wooden puzzle -push pull toys, blocks, toy telephone, puppets, finger paint

The nurse is working with a client with anxiety. An appropriate treatment goal for this client would be which of these items? Ventilate anxious feelings to a nurse Establish contact with reality Become desensitized to past trauma Learn self-help techniques

Learn self-help techniques -should enhance abilities to cope w/ anxiety

An 80 year-old resident in an assisted living facility has a temperature of 100.6 F (38.1 C). This is a sudden change from the resident's usual temperature. Which should a nurse assess first? Appetite Level of alertness Urine output Lung sounds

Level of alertness -assessing LOC will help provider determine severity of the acute temp elevation + possibility that this reps an infection -often the 1st sign of infection

The client has decreased adrenal function. What should the nursing care plan for this client include? Encourage physical activity Place the client in reverse isolation Prevent constipation Limit the number of visitors

Limit the number of visitors -avoid emotional stress of (too many) visitors -stress can put additional stress on the adrenal glands which can precipitate an addisonian crisis

A young adult seeks treatment in an outpatient mental health center. The client tells the nurse: "I am a government official being followed by spies." On further questioning, the client reveals: "My warnings must be heeded to prevent nuclear war." Which of the following actions should the nurse take? Ask for more information about the spies Listen quietly without comment Confront the client's delusion Contact the government agency

Listen quietly without comment -most therapeutic response is to listen but to also avoid being pulled into the pt's delusional system

The nurse is reviewing lab values for a client. Which abnormal serum lab value should the nurse anticipate to stay the same during hemodialysis? High creatinine Hypernatremia Hyperkalemia Low hemoglobin

Low hemoglobin -although hemodialysis improves/corrects electrolyte imbalances such as sodium, potassium + magnesium it has no effect on changing the red cells in conditions such as anemia -creatinine expected to be lowered w/ hemodialysis

The nurse is caring for a victim of domestic abuse. Which characteristic is commonly associated with the abuser? Low self-esteem High tolerance for frustration Alcohol addiction Overconfidence

Low self-esteem -even though a lot of abusers seem "tough" + "confident" they often suffer from low self-esteem

The nurse is caring for a 75 year-old client with peripheral arterial insufficiency of the lower extremities. Which intervention should be included in the plan of care to reduce leg pain? Support smoking cessation efforts Lower the legs to a dependent position Apply cold compresses Elevate the legs above the heart

Lower the legs to a dependent position -improves peripheral perfusion

A nurse admits a premature infant who has been diagnosed with respiratory distress syndrome (RDS). In planning care for the infant, the nurse understands that the pathophysiology of this disorder affects the infant's ability to do what? Regulate intrapulmonary airway pressures Maintain alveolar surface tension Adequately clear thick, sticky mucus from the lungs Stabilize thermoregulation

Maintain alveolar surface tension -central factor is the lack of normally functioning surfactant system in the alveolar sac from immaturity in lung dev b/c infant is premature

The home health nurse observes the client change an ileostomy pouch. Which action is best to help prevent skin breakdown? Change the stoma pouch daily Use deodorant soaps the contain lotion to clean the stoma Apply antiseptic cream to reddened stoma Make sure the skin around the stoma is wrinkle-free

Make sure the skin around the stoma is wrinkle-free -ensures tight, leak-free seal -power or skin prep can be applied to intact skin around stoma but not to the stoma

According to Piaget, which finding indicates that a child has attained the stage of concrete operations? Makes the moral judgment that "stealing is wrong" Reasons that homework is time-consuming yet necessary Explores the environment with the use of sight and movement Thinks in mental images or word pictures

Makes the moral judgment that "stealing is wrong" -stage of concrete operations depicted by logical thinking + moral judgments -ages 7 to 11

The client is instructed to collect stool specimens at home using the guaiac test. In addition to explaining how to collect the specimens, the nurse instructs the client to avoid certain substances prior to obtaining the stool specimens. Which of the following substances should the client avoid? (Select all that apply.) Barbecued pork chops Marinated cauliflower and broccoli Oranges Grilled sirloin steak Acetaminophen Broiled or wood-grilled salmon

Marinated cauliflower + broccoli -have high peroxidase activity Oranges -limit intake vit C b/c too much can lead to false (-) Grilled sirloin steak -avoid for @ least 3 days prior -> false (+) chicken, pork + seafood can be consumed

A 10-month old infant is admitted with a diagnosis of bacterial meningitis. Several hours after admission, during a planning conference, which of the actions suggested to the registered nurse (RN) by the practical nurse (PN) would be appropriate to add to the plan of care? Provide passive range of motion Initiate droplet precautions Measure head circumference Provide an over-the-crib protective top

Measure head circumference -assessment of neurological signs should be done frequently -hc measured b/c subdural effusions + obstructive hydrocephalus can develop as a complication of meningitis -client would have already been placed on droplet precautions + had a crib top applied to bed when he was admitted to the unit

A nurse administers cimetidine to a 79 year-old male with a gastric ulcer. Which parameter may be affected by this drug and should be closely monitored by the nurse? Mental status Liver enzymes Hemoglobin Blood pressure

Mental status -pt's > 50 or are severely ill may become temporarily confused while taking H2 receptor blockers

During medication reconciliation, the client's partner explains that the client stopped taking the prescribed sertraline and started using the partner's tranylcypromine to treat his depression. The client is now experiencing "twitching muscles" and a "racing heart." What additional adverse reaction should the nurse immediately assess for and alert the health care provider about? Atrial fibrillation Mental status changes Muscle weakness Pulmonary edema

Mental status changes -use of serotonergic agents may result in serotonin syndrome w/ confusion, nausea, palpitations, increased muscle tone w/ twitching muscles -serotonin syndrome r/t pt's taking 2 or more meds that increase CNS serotonin levels

The nurse is providing care for a client diagnosed with sickle cell crisis. Which medication ordered for pain control should be questioned by the nurse? Codeine Fentanyl (Sublimaze) Meperidine (Demerol) Morphine sulfate

Meperidine (Demerol) -not recommended in pt's w/ sickle cell dx -CNS stimulant that produces anxiety, tremors, myoclonus + generalized szs when it accumulates w/ repetitive dosing

The nurse is caring for a 68 year-old male client who had a transurethral resection of the prostate (TURP) 12 hours ago. The client has an indwelling 3-way catheter with continuous bladder irrigation. Which finding requires the nurse's immediate intervention? Reports of a feeling of discomfort from the urinary catheter Occasional suprapubic cramping about every hour Light-pink urine with a continuous stream into the collection bag Minimal drainage into the urinary collection bag

Minimal drainage into the urinary collection bag -lack of drainage needs to be reported immediately b/c minimal urinary drainage puts pt @ risk for bladder rupture -urine will be bright red from bleeding immediately after the procedure, lightening over time as bleeding decreases

An infant who has recently been diagnosed with cystic fibrosis (CF) is being assessed by the nurse. Which finding of this disease would the nurse not expect to see at this time? Moist, productive cough Meconium ileus Positive sweat test Bulky, greasy stools

Moist, productive cough -later sign of CF -noisy respirations + dry productive cough are commonly 1st respiratory signs

A 1 year-old child is receiving temporary total parental nutrition (TPN) through a central venous line. This is the first day of TPN therapy. Although all of the following nursing actions must be included in the plan of care of this child, which one would be a priority at this time? Maintain central line catheter integrity Check results of liver enzyme tests Monitor serum glucose levels Use aseptic technique during dressing changes

Monitor serum glucose levels -hyperglycemia may occur during 1st day or 2 as child adapts to high-glucose load of TPN solution

Facial puffiness, macroglossia, ptosis; coarse, sparse hair; confusion, hypothermia, bradycardia

Myxedema coma -from hypothyroidism

A family arrives at the emergency department. A parent believes the child ingested an undetermined number of acetaminophen tablets approximately 1 hour ago. The serum acetaminophen level confirms acute poisoning. Which of these orders should be implemented first? Oral activated charcoal therapy N-acetylcysteine (NAC) (Mucomyst) Ondansetron (Zofran) 0.1 mg/kg for nausea Consultation with a medical toxicologist

N-acetylcysteine (NAC) (Mucomyst) -child should be started on this antidote b/c w/o rapid intervention, acetaminophen toxicity can lead to death -activated charcoal not recommended when > 1 hr has elapsed after ingestion

An infant has just had a pyloromyotomy. Initial postoperative nursing care would include which of these approaches? Intravenous fluids for three to four days Formula or breast milk as tolerated Bland diet appropriate for age NPO then glucose and electrolyte solutions

NPO then glucose and electrolyte solutions

The client undergoes a laparoscopic removal of the appendix. Which postoperative instructions will the nurse reinforce? (Select all that apply.) No showering for 48 hours after surgery Maintain bedrest for 24 hours before gradually resuming regular activities Restrict diet to bland, easily digestible food for a few days Gently scrub off the "skin glue" when you feel able Some shoulder discomfort can be expected Use 2 tablespoons of Milk of Magnesia if no bowel movement 3 days after surgery

No showering for 48 hours after surgery -keep dressings clean + dry Restrict diet to bland, easily digestible food for a few days Some shoulder discomfort can be expected -laparoscopic surgery involves using carbon dioxide gas to open inside of abd which pushes up diaphragm - may cause shoulder discomfort Use 2 tablespoons of Milk of Magnesia if no bowel movement 3 days after surgery

A 7 year-old child is hospitalized for acute glomerulonephritis. Which nursing action is a priority on the plan of care? Assess for generalized edema Encourage rest during hyperactive periods Monitor for increased urinary output Note patterns of increased blood pressure

Note patterns of increased blood pressure -priority is the eval for HTN b/c in the course of the dx, high BP has most potential for complications

The nurse is caring for a client taking antipsychotic drugs. Why is it important for the nurse to monitor blood pressure in this client? Orthostatic hypotension is a common side effect Rising trends in blood pressure will indicate when an antiparkinsonian drug is needed Most antipsychotic drugs cause elevated blood pressure Blood pressure will determine if dietary restriction of sodium is needed

Orthostatic hypotension is a common side effect

The staff nurse prepares an 88 year-old female for discharge and confirms that follow up care for a home health nurse is scheduled. During the medication reconciliation process, the client's husband states he hopes that his wife has "learned her lesson" and will take her medications as ordered. What action by the staff nurse is indicated? (Select all that apply.) Notify the home health agency nurse about the husband's statement Notify the health care provider of the need to delay discharge Ask the husband to manage his wife's medication at home Ask the husband why he thinks his wife is not taking her medications properly Write a schedule with days and times for the client to take her medication

Notify the home health agency nurse about the husband's statement Ask the husband why he thinks his wife is not taking her medications properly -nurse should 1st determine what husband meant + directly ask why he thinks his wife is not taking her meds properly Write a schedule with days and times for the client to take her medication

Check blood pressure Follow up on lab values before the visit Observe client findings for the effectiveness of antibiotic therapy Review oral intake with the client

Observe client findings for the effectiveness of antibiotic therapy -determine whether antibiotic therapy has been effective in treating this serious kidney infection

A 62 year-old arrives in the emergency department reporting chest pain. Following protocol, the client is given nitroglycerin and aspirin. The initial ECG indicates nondiagnostic changes in ST segment or T waves. What does the nurse anticipate will happen next? Prep the client for percutaneous coronary intervention (PCI) Begin oxygen at 4L/min per nasal cannula Administer IV morphine Obtain initial cardiac marker levels and other labs

Obtain initial cardiac marker levels and other labs -normal or nondiagnostic changes in ECG rep low risk of MI -there's no indication that pt's pain is not being controlled by nitroglycerin -no need for a PCI w/o confirmation of a MI

A client with schizophrenia receives haloperidol 5 mg three times a day. The client's family is alarmed and calls the clinic nurse when "his eyes rolled upward." The nurse should recognize this finding as what type of side effect? Nystagmus Tardive dyskinesia Dysphagia Oculogyric crisis

Oculogyric crisis -refers to involuntary muscle spasm of the eye

A nurse is caring for a 14 month-old just diagnosed with cystic fibrosis. The parents state this is the first child in either family diagnosed with this disease, and ask about the risk to future children. What is the best response by the nurse? One in four risk for each child to have the disease One in four chance for each child to carry that trait One in two chance that each child will have the disease One in two chance of avoiding the trait and disease

One in four risk for each child to have the disease -autosomal recessive -both parents must be carriers -for each pregnancy: -25% chance of child having dx, 50% chance of carrying trait + 25% of having neither trait or dx

During family counseling, the parents display the dysfunctional phenomenon called triangulation. Which of the following behaviors demonstrates triangulation? One parent accuses the other of spending all her free time and giving special favors to the child The parents state that the grandparents and several other relatives will need to be consulted before any decisions are made One parent admits to being disappointed about not having another child and is now volunteering at a children's center

One parent accuses the other of spending all her free time and giving special favors to the child -triangulation = dysfunctional phenomenon where tension b/w a dyad (2 people who have a relationship) is displaced onto a 3rd person (child, friend, etc)

The nurse is reviewing the laboratory results for several clients. Which of the laboratory result indicates a client with partly compensated metabolic acidosis? Chloride 100 mEq/L (100 mmol/L) Sodium 130 mEq/L (130 mmol/L) Hemoglobin 15 g/dL (150 g//L) PaCO2 30 mm Hg

PaCO2 30 mm Hg (normal is 35-45) -Hgb WNL

The 72 year-old client, admitted for exacerbation of chronic obstructive pulmonary disease (COPD), is receiving 2 liters of oxygen per nasal cannula but is reporting dyspnea. An arterial blood gas (ABG) test is ordered and the results are: PaO2 40, pH 7.38, PaCO2 50, HCO3 28. Which option best explains the finding and indicates the required treatment? PaO2 is too high and oxygen flow rate should be decreased PaO2 is too low and oxygen flow rate should be increased PaCO2 is too high and deep breathing should be encouraged PaO2 is within normal limits and the oxygen flow rate should not be changed

PaO2 is too low and oxygen flow rate should be increased

The nurse is caring for a 10 month-old infant diagnosed with iron-deficiency anemia. Based on this diagnosis, which of these findings should the nurse anticipate? Hemoglobin level of 12 g/dL Pale mucosa of the eyelids and lips A heart rate between 80 and 130 Poor appetite

Pale mucosa of the eyelids and lips

A nurse is talking by telephone with a parent of a 4 year-old child who has chickenpox. Which approach demonstrates appropriate teaching by the nurse? Chewable aspirin is the preferred analgesic Recommend an antiviral medication to relieve itching The illness is only contagious when the lesions are present Papules, vesicles and crusts will be present at one time

Papules, vesicles and crusts will be present at one time -all 3 stages of the chickenpox lesions will be present on kid's body @ same time -person w/ chicken pox contagious 1-2 days before blisters appear + remain contagion until all the blisters have crusted over

A 9 year-old is taken to the emergency department with right lower quadrant pain and vomiting. During the preparation of the child for an emergency appendectomy, what should the nurse expect to be the child's greatest fear? Change in body image Perceived loss of control An unfamiliar environment Guilt over being hospitalized

Perceived loss of control -for school aged children, major fears are loss of control + separation from friends/peers

The nurse is eating in the hospital cafeteria when a toddler at a nearby table chokes on a piece of food and turns slightly blue. What would be the most appropriate initial action taken by the nurse? Perform abdominal thrusts Begin mouth to mouth resuscitation Call for the emergency response team Give the child water to help in swallowing

Perform abdominal thrusts -@ this age the most effective way to clear the airway of food is to perform abd thrusts

While caring for a client in the 2nd stage of labor, the health care provider requests 1% lidocaine, a 10 mL syringe and a 23 gauge 1.5 inch needle. What does the nurse anticipate that the health care provider is preparing to do? Perform an episiotomy Administer an epidural Initiate oxytocin infusion Correct a prolapsed cord

Perform an episiotomy -epidurals typically administered during 1st stage of labor

The client is diagnosed with Clostridium difficile (C. difficile.) The nurse, who does not apply personal protective equipment (PPE), enters the room to administer a medication. The client requests assistance to sit up in bed before taking the medication. What is the next action by the nurse? Leave the room to apply PPE Assist the client to move up in bed Reposition the client and then apply alcohol-based hand rub Perform hand hygiene with soap and water

Perform hand hygiene with soap and water -in this scenario, nurse needs to 1st wash hands w/ soap + water + then apply PPE before providing any care

A client is admitted to the psychiatric unit with a diagnosis of bipolar disorder. This client constantly "bothers" other clients, tries to help the housekeeping staff, demonstrates pressured speech and demands constant attention from the staff. Which activity should the nurse attempt to get the client to do? Checkers Reading Ping-pong Cards

Ping-pong -provides an outlet for physical energy + requires limited attention -other options would over-tax pt's level of self-control b/c pt has a need to be active

A 3 year-old child is brought to the health clinic. The grandmother reports that the child is always "scratching his bottom" and is "extremely irritable." Based on this information, which health issue would the nurse assess for initially? Ringworm Scabies Allergies Pinworm

Pinworm (the "P in worms" are found where "pooh" comes out) -findings include intense perineal itching (usually worse @ night -> poor sleep patterns, generalized irritability, restlessness, bedwetting, short attention span) -eggs will stick to piece of clear cellophane tape placed against rectum -samples can also be taken from under kid's nails -ringworm is a fungus w/ characteristic round, itchy irritations on skin

During the morning rounds, a nurse observes that a client diagnosed with heart failure has developed sudden anxiety, diaphoresis and dyspnea. The nurse auscultates crackles bilaterally. Which nursing intervention should be performed first? Administer the PRN IV morphine Contact the health care provider Incorrect Take the client's vital signs Place the client in a sitting position with legs dangling

Place the client in a sitting position with legs dangling -to pool blood in legs -helps to diminish venous return to the heart + minimize pulmonary edema + helping pt breathe more easily

A 10 year-old child is recovering from a splenectomy after a traumatic injury. The child's laboratory results show a hemoglobin of 8.8 g/dL and a hematocrit of 26%. What is a priority approach that the nurse should include in the plan of care? Plan for regularly scheduled rest periods Promote a diet rich in iron and lean red meats Restrict the consumption of carbonated beverages Encourage bed activities and games for the next five days

Plan for regularly scheduled rest periods -initial priority for this pt is rest d/t lack of sufficient RBCs to carry O2

A woman is hospitalized for treatment of pregnancy induced hypertension (PIH) in the third trimester. She is now receiving magnesium sulfate intravenously. The nurse understands that this medication is used mainly for what purpose? Increase uterine blood flow Maintain normal blood pressure Prevent preeclamptic seizures Decrease the respiratory rate

Prevent preeclamptic seizures

The nurse is caring for a client with orders for complete bed rest. Which action by the nurse is most important in the prevention of the formation of deep vein thrombosis (DVT)? Elevate the foot of the bed Encourage isometric leg muscle exercises Prevent pressure at back of the knees Apply knee high support stockings

Prevent pressure at back of the knees -will minimize venous stasis + DVT

A client is admitted to the hospital with findings of liver failure with ascites. A health care provider orders spironolactone. What is the pharmacological effect of this medication? Depletes potassium reserves Promotes sodium and chloride excretion Increases aldosterone levels Combines safely with antihypertensives

Promotes sodium and chloride excretion -spares potassium levels

A nurse is administering lidocaine to a client with a myocardial infarction. Which assessment finding requires the nurse's immediate action? Central venous pressure reading of 11 Blood pressure of 144/92 Pulse rate of 48 beats per minute Respiratory rate of 22

Pulse rate of 48 beats per minute -one of the s/e of lidocaine is bradycardia -med shouldn't be admin w/o continuous cardiac monitoring

A nurse is caring for a client with a new order for bupropion hydrochloride for treatment of depression. The order reads "Wellbutrin 175 mg twice a day for four days." What is the appropriate action? Observe the client for mood swings Question this medication dose Give the medication as ordered Monitor neurologic signs frequently

Question this medication dose -bupropion should be started @ 100 mg 2x/day for 3 days then increased to 150 mg 2x/day

A 16 year-old adolescent is admitted for Ewing's sarcoma of the tibia. In discussing the care with the parents, the nurse should understand that the initial treatment for this diagnosis usually includes which approach? Amputation above the tumor Surgical excision of the mass Radiation with adjunctive chemotherapy Bone marrow graft in the affected leg

Radiation with adjunctive chemotherapy -to reduce size of tumor

The nurse measures the head circumference and chest circumference of a 18 month-old infant. The nurse compares the two measurements to each other and notes they are approximately the same. What action should the nurse take? Notify the health care provider Palpate the anterior fontanel Record the findings in the chart Feel the posterior fontanel

Record the findings in the chart -expected findings -b/w 6 months + 2 yrs an infant's hc + cc measurements are about the same

A nurse is examining an infant in a clinic. Which nursing assessment for the infant is most valuable in the identification of serious visual defects? Pupil response to light Cover test Visual acuity Red reflex test

Red reflex test -brilliant, uniform red reflex important sign b/c it virtually rules out almost all serious defects of the cornea, aqueous chamber, lens + vitreous chamber

A newborn who is delivered at home and without a birth attendant is admitted to the hospital for observation. The initial temperature is 95 F (35 C) axillary. The nurse should recognize that cold stress may lead to what complication? Hyperglycemia Metabolic alkalosis Lowered basal metabolic rate Reduced partial pressure of oxygen in arterial blood (PaO2)

Reduced partial pressure of oxygen in arterial blood (PaO2)

The nurse is monitoring a client who is receiving the thrombolytic agent alteplase for treatment of an acute myocardial infarction (AMI). What outcome indicates the client is receiving adequate therapy within the first few hours of treatment? Cardiac enzymes are within normal limits Absence of cardiac arrhythmias Reduction of ST-segment elevation on a 12-lead ECG Stabilization of blood pressure

Reduction of ST-segment elevation on a 12-lead ECG -alteplase is used in the mgmt of AMI w/ ST segment elevation (STEMI)

A client has a history of chronic obstructive pulmonary disease (COPD). The nurse enters the client's room to find that the nasal cannula is in proper position with the oxygen set at 6 liters per minute, the client's color is flushed and the respirations are 8 per minute. What should the nurse do first? Lower the oxygen's flow rate Place client in a higher sitting position Remove the nasal cannula for at least five minutes Check the client's pulse for strength and rate

Remove the nasal cannula for at least five minutes -pt has findings of O2 toxicity nurse should then: -pulse assessment -change of position -lower O2 flow rate

A newly admitted client reports taking phenytoin for several months. Which of the following assessments should the nurse be sure to include in the admission report? (Select all that apply.) Report of any seizure activity Serum phenytoin levels Report of unsteady gait, rash and diplopia Report of anorexia, numbness and tingling of the extremities

Report of any seizure activity Serum phenytoin levels Report of unsteady gait, rash and diplopia

A client with an IV antibiotic infusing is scheduled to have blood drawn at 1:00 pm for a peak antibiotic level measurement. The nurse notes that the IV infusion is running behind schedule and won't be infused until 1:30 pm. What action should the nurse take? Increase the infusion rate to finish it by 1:00 pm Notify the client's health care provider Reschedule the laboratory test for 2:00 pm Stop the infusion at 1:00 pm and get the blood drawn

Reschedule the laboratory test for 2:00 pm -if an antibiotic infusion will not be completed @ the time the peak blood level is scheduled to be drawn, a nurse should ask the blood sampling time to be adjusted -peak level should be drawn 30-60 minutes after completing infusion

The nurse is assessing a toddler diagnosed with croup. Which initial finding requires the nurse's immediate attention? Apical pulse of 130 Lethargy for the past hour Respiratory rate of 54 Coughing up copious secretions

Respiratory rate of 54 -s/s of impending airway obstruction

The home care nurse is teaching the client about managing heart disease at home. What lifestyle change will promote comfort and potentially help prevent a medical crisis and unwanted hospitalization? Relax and contract leg muscles Record and monitor daily weight Participate in a progressive exercise routine Rest in an armchair instead of lying in bed

Rest in an armchair instead of lying in bed -upper body should be elevated -resting in an armchair better than lying in bed b/c it decreases cardiac workload + facilitates breathing -monitoring I + O doesn't promote comfort

A parent brings a 3 month-old infant into the clinic, reporting that the child seems to be spitting up all the time and has a lot of gas. The nurse expects which findings on the initial history and physical assessment? Diarrhea and poor skin turgor Increased sleeping and listlessness Restlessness and irritability Increased temperature and lethargy

Restlessness and irritability -can develop if an allergy is present

A client with chronic obstructive pulmonary disease (COPD) and a history of coronary artery disease is receiving aminophylline 25 mg/hour. Which finding would be associated with side effects of this medication? Decreased blood pressure and respirations Flushing and headache Increased heart rate and blood pressure Restlessness and palpitations

Restlessness and palpitations -bronchodilator -r/t chemically to caffeine

The nurse is monitoring oxygen saturation levels of preterm infants in the neonatal intensive care unit (NICU). Which of the following complications of oxygen therapy in preterm infants is the most common? Necrotizing enterocolitis Hyperbilirubinemia Retinopathy of prematurity (ROP) Bronchopulmonary dysplasia (BPD)

Retinopathy of prematurity (ROP) -prolonged exposure to high concentrations of O2 will cause irreversible damage to the eyes of preterm infants -BPD develops as a result of an infant's lungs becoming irritated or inflamed by mechanical ventilation, high levels O2, infections, heredity

A nurse is teaching a class for new parents at a local community center. Which activity would the nurse stress as being the most hazardous for an 8 month-old? Jumping on a bed Playing around electrical outlets Riding in a car Eating whole peanuts

Riding in a car -car accidents are a leading cause of death in babies + children -drowning is 2nd most common cause of death among children

Red rash becomes ring-shaped, with a red-colored, raised border and clearer center

Ringworm (tinea corporis) -contagious fungal infection lives on, not in skin

A school nurse is called to the playground for an episode of mouth trauma. The nurse finds that the front tooth of a 9 year-old child has been avulsed ("knocked out"). After recovering the tooth, the initial response by the nurse should be to do what with the tooth? Hold the tooth by the roots until reaching the emergency room Rinse the tooth in water before placing it into its socket Place the tooth in a clean plastic bag for transport to the dentist Ask the child to replace the tooth even if the bleeding continues

Rinse the tooth in water before placing it into its socket -important to rinse the dirty tooth in water, saline solution or milk before re-implantation -if possible, replace tooth into its socket w/in 30 minutes while avoiding contact w/ root -child should be taken to dentist asap

The nurse is caring for a 50 year-old client diagnosed with advanced cirrhosis of the liver. Which nursing diagnosis should take priority? Fluid volume excess: ascites Risk for injury: hemorrhage Altered nutrition: less than body requirements Risk for injury related to peripheral neuropathy

Risk for injury: hemorrhage -liver dx interferes w/ production of prothrombin + other factors essential for blood clotting

The nurse is caring for a client with a diagnosis of gastroesophageal reflux disease (GERD). The primary health care provider's orders include omeprazole (Prilosec) twice a day, Maalox prior to meals, elevation of the head of the bed, acid-reflux diet, and no alcohol. Which one of these orders would the nurse question? Prescribed diet Schedule for antacid Bed position Schedule for the proton-pump inhibitor

Schedule for antacid -schedule for antacids should be 1-3 hrs after eating + @ bedtime as needed

A nurse is caring for a client who is receiving methyldopa. Which assessment finding would indicate to the nurse that the client may be having an adverse reaction to the medication? Nausea Palpitations Sedation Hyperkalemia

Sedation -methyldopa used to treat HTN -nurse should assess pt for alterations in mental status such as sedation

A nurse is planning care for a 2 year-old hospitalized child. Which issue will produce the most stress at this age? Loss of control Separation anxiety Bodily injury Fear of pain

Separation anxiety -peaks during toddler years

A 5 year-old child is rushed to the emergency department approximately six hours after ingesting an undetermined amount of acetaminophen. Which lab test should receive priority attention by the nurse? Alanine transaminase test (ALT) and aspartate transaminase test (AST) Prothrombin time (PT) and INR Electrolytes and blood urea nitrogen (BUN) Serum acetaminophen concentration (APAP)

Serum acetaminophen concentration (APAP) -will help determine N-acetylcysteine therapy (the antidote) ALT + AST r/t extent of liver damage

A client with a history of heart disease takes prophylactic aspirin daily. What should the nurse monitor to help prevent aspirin toxicity? Serum albumin Protein intake Conductive hearing loss Serum potassium

Serum albumin -aspirin + salicylic acid bound to serum albumin -low serum albumin level may result in altered salicylate binding, thereby increasing the availability of unbound (active) drug for toxic effects -conductive hearing loss typically the result of ear infections, allergies, or tumors - not aspirin toxicity

The parents of a 3 year-old ask the nurse about preventing injuries. What is one of the most effective methods caregivers can use to teach young children about injury prevention? Set good examples themselves Protect the child from outside influences Make sure the child understands the safety rules Discuss the consequences of not wearing protective devices

Set good examples themselves -preschoolers often imitate behaviors in others + they are quick to notice discrepancies b/w what they see + what they are told to do

The nurse is caring for a client diagnosed with major depressive disorder. The nurse understands that this client is at highest risk for a suicide attempt at which of these times? Seven to 14 days after initiation of antidepressant medication and psychotherapy When the client is removed from the security room and placed in an individual room One to two days after admission to an inpatient facility Within 24 to 48 hours following an angry outburst with the family

Seven to 14 days after initiation of antidepressant medication and psychotherapy -as the depression lessens, these pt's often have energy to implement their plan of suicide

The nurse works in an inpatient psychiatric setting. What would be the best reason why the nurse should limit touch to a handshake with a client? Touching a client can set off a violent episode Refusing to touch a client indicates a lack of concern Shaking hands allows the use of touch in a professional manner Clients may misconstrue touch as an invitation to more intimate behavior

Shaking hands allows the use of touch in a professional manner -in psych setting extent of physical contact should be limited to handshakes

The nurse is caring for a client in the late stages of amyotrophic lateral sclerosis (ALS). Which finding would the nurse expect? Confusion Shallow respirations Tonic-clonic seizures Loss of half of visual field

Shallow respirations -ALS does not impair a person's mind or intelligence -does not affect person's ability to see, smell, taste, hear or recognize touch

A client with a panic disorder has a new prescription for alprazaolam. In teaching the client about the drug's actions and side effects, which point should the nurse emphasize? The medication acts as a stimulant Initial side effects often continue unchanged Dosage will be increased as tolerated Short-term relief can be expected

Short-term relief can be expected -alprazolam is a short-acting benzo used to quickly control panic s/s

A client states, "I feel funny." The nurse uses electronic equipment to obtain vital signs and notes these findings: blood pressure 100/56 mm Hg, pulse 38, respirations 26. The client's previous reading: blood pressure 130/88 mm Hg, pulse 82, respirations 21. List the correct order of actions the nurse should now take (with 1 being the top priority). Assess for chest pain, dyspnea, low oxygen saturation, restlessness or other signs of respiratory or cardiac impairment Simultaneously check an apical and radial pulse manually Notify the health care provider Anticipate the need for ECG, oxygen administration, and emergency pacing

Simultaneously check an apical and radial pulse manually -@ this point pt not in crisis but needs a quick targeted assessment Assess for chest pain, dyspnea, low oxygen saturation, restlessness or other signs of respiratory or cardiac impairment Notify the health care provider Anticipate the need for ECG, oxygen administration, and emergency pacing

The nurse has been teaching a client diagnosed with heart failure about proper nutrition. Which of these lunch selections indicates that the client has learned about sodium restriction? Cheeseburger and baked potato with butter Sliced turkey sandwich with a side of canned pineapple Cheese sandwich with a glass of 2% milk Mushroom pizza and ice cream made from whole milk

Sliced turkey sandwich with a side of canned pineapple -sliced turkey not a highly processed food -canned fruits low in sodium

A client diagnosed with hypertension is started on atenolol. The nurse should instruct the client to immediately report which of these findings? Feeling tired Slow, irregular pulse Decreased sex drive Insomnia

Slow, irregular pulse

A nurse is caring for an acutely ill 10 year-old child. Which assessment finding would require the nurse's immediate attention? Rapid bounding pulse Profuse diaphoresis Temperature of 101.3 F (38.5° C) Slow, irregular respirations

Slow, irregular respirations -slow + irregular RR is a sign of respiratory fatigue + failure in an acutely ill child

A client is being discharged with a prescription for chlorpromazine. Before the client leaves for home, which finding should the nurse teach the client to report right away? Change in libido and breast enlargement Abdominal pain and nausea Lethargy and drooling Sore throat and fever

Sore throat and fever -may be findings of agranulocytosis (serious s/e)

A nurse is caring for a client suspected to have a diagnosis of active tuberculosis (TB). Which diagnostic tests is essential for the nurse to obtain for the determination of the presence of active TB? Tuberculin skin testing White blood cell count Chest x-ray anterior/posterior and lateral Sputum culture for cytology

Sputum culture for cytology -method for determination if active TB is present -takes 1-2 wks to get results

The nurse is using the Glasgow Coma Scale to assess a client diagnosed with a traumatic brain injury. When the client does not obey verbal commands to move, which technique should the nurse use to evaluate motor function? Squeeze the trapezius muscle firmly Rub the sternum with the knuckles Lift the client's arm and observe for pronation and drift Apply finger tip pressure for 10 seconds

Squeeze the trapezius muscle firmly -stronger central pressure needed to assess motor function when pt does not obey commands to move -if pt's eyes do not open spontaneously or to sound, the nurse will assess eye opening using physical peripheral pressure such as finger tip pressure

The nurse assesses delayed gross motor development in a 3 year-old child. The inability of the child to do which action confirms this finding? Catch a ball Ride a bicycle Stand on one foot Skip on alternate feet

Stand on one foot

The nurse is providing information to a client with diarrhea. Which of the following food choices should the client be advised to avoid? Pulp-free fruit juice Macaroni made from white or refined flour Steel-cut oatmeal with nuts and dried fruit Tender, well-cooked meat

Steel-cut oatmeal with nuts and dried fruit -nuts, dried fruit, whole grains + highly processed or fatty meats should be avoided

The nurse is caring for a client with total parenteral nutrition (TPN). What is the most important action on the part of a nurse? Record the number of stools per day Sterile technique for dressing change at IV site Maintain strict intake and output records Monitor for cardiac arrhythmias

Sterile technique for dressing change at IV site -pt's receiving TPN very susceptible to infection -the concentrated glucose solutions are a good medium for bacterial growth

There's an order to check the pH of aspirate every four hours for a client who has a continuous tube feeding. The nurse checks the aspirate at the designated time and the pH is 8. What action should the nurse take? Irrigate the tube with water and reassess pH Stop the tube feeding for about an hour and then reassess aspirate Continue the tube feeding as scheduled Hold the tube feeding and notify the health care provider

Stop the tube feeding for about an hour and then reassess aspirate -higher pH indicates either small intestine placement or even in lungs

The nurse is preparing a client for a kidney, ureter, bladder (KUB) radiograph test. Prior to the test, which action should the nurse plan to take for this client? Plan to have a fleets enema given prior to the examination Incorrect Medicate the client with a PRN antihistamine prior to the examination Take no special actions before this examination Keep the client NPO for eight hours before the examination

Take no special actions before this examination -it's a simple xray

The client is prescribed dexamethasone by mouth every other day and asks the nurse for more information about the medication. What information would the nurse want to share with the client? (Select all that apply) Take the medication with food Do not get any immunizations or skin tests Mark your calendar to keep track of doses Take the medication at bedtime Avoid dairy products

Take the medication with food -to avoid stomach upset -better taken in am Do not get any immunizations or skin tests Mark your calendar to keep track of doses

A nurse is caring for a client with a serum potassium of 3.2 mEq/L. The client is placed on a cardiac monitor and started on IV infusion of 40 mEq KCL in 1000 mL of 5% dextrose in water. Which ECG findings indicate that the infusion of potassium should be stopped? Shortened PR interval Narrowed QRS complex Tall, peaked T waves Prominent U waves

Tall, peaked T waves -finding in hyperkalemia

The nurse is caring for a client diagnosed with anemia and confusion. Which task could the nurse delegate to the unlicensed assistive personnel (UAP)? Assess and document skin turgor and skin color changes Suggest foods that are high in iron and prepare a list of likes and dislikes Test stool for occult blood and urine for pH with a report of the results Report mental status changes and level of mental clarity

Test stool for occult blood and urine for pH with a report of the results

A healthy 18 year-old is entering college in the fall. Which immunization would the health care provider recommend prior to college? (Select all that apply.) Pneumococcal polysaccharide vaccine (PPSV23) Shingles vaccine Tetanus, Diphtheria, Pertussis vaccine (Tdap) Human papillomavirus (HPV) vaccine Meningococcal conjugate vaccine (MCV4) Seasonal influenza vaccine

Tetanus, Diphtheria, Pertussis vaccine (Tdap) Human papillomavirus (HPV) vaccine Meningococcal conjugate vaccine (MCV4) Seasonal influenza vaccine adults > 50 get shingles vaccine PPSV23 given to adults > 65

Nursing students are reviewing the various types of oxygen delivery systems. Which oxygen delivery system is the most accurate? The Venturi mask A partial nonrebreather mask The simple face mask A nasal cannula

The Venturi mask -high flow device that entrains room air into a reservoir device on the mask + mixes room air w/ 100% O2

The nurse is teaching a group of parents about poisoning risks in the home and how to report information in the event of an accidental poisoning. The nurse should instruct the parents to first state the ingested substance and then to state which of these points of information? The estimated time of the accidental poisoning with a confirmation to bring the ingested substance containers The parent's name and telephone number The currency of the immunization and allergy history of the child The affected child's current age and most recent weight

The affected child's current age and most recent weight -this gives the HCP an opportunity to calculate + prepare the needed dosage for an antidote in preparation for when the child arrives

A client with asthma has low-pitched wheezes present on the final half of exhalation. One hour later the wheezing is higher pitched and extends throughout exhalation. What should this change in assessment finding indicate to the nurse? The client is hyperventilating The client needs to be suctioned The airway obstruction has improved The airway obstruction has increased

The airway obstruction has increased -the higher pitched a sound is, the more narrow the airway

A report of 10 pounds weight loss in the last month A report of the sudden onset of irritability in the past two weeks A comment by the client: "I just can't sit still." The appearance of eyeballs that appear to "pop" out of the client's eye sockets

The appearance of eyeballs that appear to "pop" out of the client's eye sockets -exophthalmos -can result in corneal abrasion w/ severe eye pain or damage when eyelid unable to blink down over protruding eyeball -eye drops or ointment may be needed

A nurse uses the New Ballard Scale to assess gestational age of a newborn. The assessment score total is very high. What is a reasonable interpretation of this result? The baby experienced distress during labor The baby is large for gestational age The baby is premature The baby is post-term

The baby is post-term high score = post maturity low score = premies fetal distress during labor can result in lower scores

There is an order to insert a urinary catheter. The client is an adult female. The nurse slips the catheter approximately 4-5 inches (10-12 cm) into an opening, but no urine is obtained. What is the most probable reason for this outcome? The catheter is located in the vaginal canal No urine is present in the bladder The bladder is overdistended without the ability to empty The catheter is not inserted far enough into the canal

The catheter is located in the vaginal canal -for an adult female, urinary cath is inserted about 2-3 inches in urinary meatus until urine flow begins

The child diagnosed with central diabetes insipidus (DI) is being treated with desmopressin nasal. What information is important to reinforce with the family? (Select all that apply.) The child should wear MedicAlert® identification Using the nasal preparation may cause a stuffy nose A parent or other responsible adult should supervise and help the child use the medication Muscle weakness, spasms or cramps are expected and harmless effects of the medication The medication increases urine production It is important to decrease intake of water and other fluids while taking this

The child should wear MedicAlert® identification Using the nasal preparation may cause a stuffy nose A parent or other responsible adult should supervise and help the child use the medication It is important to decrease intake of water and other fluids while taking this

The nurse works in an ambulatory care clinic where there are four children with gastrointestinal findings waiting to be seen by the health care provider. Which child is at greatest risk for developing metabolic acidosis? The child who has been vomiting for more than 48 hours The child with nausea and anorexia The child with severe diarrhea for 24 hours The child with alternating constipation and diarrhea

The child w/ severe diarrhea for 24 hours -can lead to excessive loss of sodium bicarb from body prolonged vomiting -> metabolic alkalosis (d/t acid loss)

The nurse receives a report on a client being admitted with the diagnosis of cirrhosis of the liver and ascites. What should the nurse emphasize to the nursing assistant about providing care for this client? The client may ambulate and sit in a chair as tolerated The client should remain on bed rest in a semi-Fowler's position The client is to ambulate as tolerated and be positioned in semi-Fowler's position when in bed The client should ambulate as tolerated, resting in bed with legs elevated between walks

The client should ambulate as tolerated, resting in bed with legs elevated between walks -will mobilize edema + ascites

During a situation of pain management, which statement is a priority to consider for the ethical guidance of a nurse? Clients have the right to have their pain relieved Cultural sensitivity is fundamental to pain management The client's self-report is the most important consideration Nurses should not prejudge a client's pain using their own values

The client's self-report is the most important consideration -most important consideration -other statements correct but not most important consideration

The nurse is caring for a 10 year-old child who is diagnosed with diabetes insipidus (DI) and is receiving vasopressin. What is the priority for the nurse to teach the child and the family members about this prescribed medication? Parents should administer the daily intramuscular injections The child will need intravenous therapy for several weeks The family must observe the child for dehydration The family must monitor the child for arrhythmias

The family must monitor the child for arrhythmias

An 18 month-old child is on peritoneal dialysis in preparation for a renal transplant in the near future. When a nurse obtains the child's health history, the mother indicates that the child has not had the first measles, mumps, rubella (MMR) immunization. What should the nurse understand about the child's situation and administration of the immunization? Live vaccines are withheld in children with renal chronic illness The risk of the vaccine side effects precludes the administration of the vaccine An inactivated form of the vaccine can be given at any time The measles, mumps and rubella (MMR) vaccine should be given now, before the transplant

The measles, mumps and rubella (MMR) vaccine should be given now, before the transplant

The nurse is making rounds with the pediatrician on the postpartum unit. Which of the following newborns should the pediatrician see first? The newborn with widely spaced cranial suture lines The newborn delivered sixteen hours ago, who has yet to pass the first meconium stool The newborn, delivered eight hours ago, whose clamped umbilical cord has two arteries and one vein The term infant whose blood glucose is 50 mg/dL

The newborn with widely spaced cranial suture lines -should be palpable and separated -if they are widely spaced it may be an indication of hydrocephaly or growth restriction infants usually pass 1st meconium w/in 12-24 hrs after birth

A client is diagnosed with iron-deficiency anemia. What is the cause of the symptoms associated with this condition? Reduced oxygen saturation Tissue hypoxia Destruction of red blood cells (RBCs) Decreased cardiac output

Tissue hypoxia -result of not having enough functioning Hgb in the blood to oxygenate the tissues

The nurse works with children who have chronic conditions requiring frequent hospitalization and activity limitations. Which statement best describes the effects of immobility in children? Children are more susceptible than adults to the multisystem effects of immobility The physical effects of immobility are similar in both children and adults Immobility promotes independence and self-reliance in children Immobilized children quickly develop confusion and mental status changes

The physical effects of immobility are similar in both children and adults

The nurse is having a discussion with the parents of a newborn who was diagnosed with hypospadias. The nurse should communicate which point? The initial repair is delayed until six to eight years of age The postoperative appearance of the penis will be normal The surgery may be performed in stages over a period of time Circumcision can be performed at any time

The surgery may be performed in stages over a period of time -corrected in stages as soon as the child can tolerate surgery + before child turns school age

The nurse is talking with the parents of a child who has recently been diagnosed with Hemophilia A. What should the nurse understand about the offspring of an unaffected father and a carrier mother? It is likely that all sons will be affected There is a 25% chance a daughter will be a carrier There is a 50% probability that sons will have the disease Incorrect Every daughter is likely to be a carrier

There is a 25% chance a daughter will be a carrier -carrier woman only has 1 affected x chromosome, which is why there's only a 25% chance of a son having hemophilia watch out for the words "every" and "all"

The nurse is talking with the family of an 18 month-old toddler who is newly diagnosed with retinoblastoma. Which point is a priority when discussing this diagnosis with the parents? There is a need for genetic counseling Prepare them for their child's permanent disfigurement Suggest that total blindness may follow surgery Inform them that even aggressive treatment is usually ineffective

There is a need for genetic counseling -rare CA that runs in families + there is a high risk that future offspring will be affected

A client is receiving nitroprusside intravenously for the treatment of acute heart failure with pulmonary edema. What diagnostic lab value should the nurse monitor during the administration of this medication? Blood urea nitrogen Thiocyanate Potassium level Arterial blood gasses

Thiocyanate -nitroprusside metabolism involves production of cyanide (CN) - which may be extremely toxic -cyanide normally converted to thiocyanate + is eliminated by kidneys

The nurse assesses several postpartum women. Which of these women is at the highest risk for a puerperal infection? Three days postpartum, temperature is 100.8 (38.2 C) for two days after undergoing cesarean section Twelve hours postpartum following vaginal delivery, temperature is 100 (37.7 C) Seven days postpartum, temperature is 99 F (37.2 C) since vaginal delivery Five days postpartum, temperature is 99.6 F (37.6 C) since undergoing cesarean section

Three days postpartum, temperature is 100.8 (38.2 C) for two days after undergoing cesarean section -temp of 100.4 or > on 2 successive days (not counting 1st 24 hrs after birth) = infection -risk of endometritis increases after c/s delivery

The nurse assesses a client who has been taking haloperidol for several months. Which adverse effect must be immediately reported to the health care provider? Dry, harsh cough Constipation Tongue thrusting and facial grimacing Muscle flaccidity

Tongue thrusting and facial grimacing -dystonic movements - have potential of becoming irreversible muscle rigidity not muscle flaccidity

A client has been taking alprazolam for three days. The nurse should expect to find which intended effect of this drug? Relief of insomnia and phobias Increased coordination and ability to concentrate Sedation and long-term analgesia Tranquilization and calming effects

Tranquilization and calming effects -alprazolam is a benzo used in the tx of anxiety, panic disorder, + anxiety associated w/ depression -this med is a CNS depressant, producing drowsy or calming effect

The nurse monitors a client after intravenous atropine sulfate was used to treat bradycardia. Which finding should immediately be reported to the health care provider? Urinary hesitancy Transient arrhythmias Dry mouth Blurred vision

Transient arrhythmias -most common adverse effects (dry mouth, blurred vision, urinary hesitancy) - usually reversible when therapy d/c

A nurse is assigned to care for a client who has been diagnosed with an intracranial aneurysm that has now stopped leaking. To minimize the risk of another bleeding episode, or rupture, the nurse should plan to take which of these actions? Apply a warming blanket for temperatures of 98 F (36.6 C) or less Treat any elevation in blood pressure Keep the client in a upright sitting position Avoid arousal of the client except for family visits

Treat any elevation in blood pressure

A 40 year-old male, with a history of intravenous drug use, is seen in the emergency department with severe myalgia and a petechial rash. The initial diagnosis is infective endocarditis (IE). Which approach would be the priority test for diagnosing IE? Chest x-ray Echocardiogram Complete blood count and coagulation panel Two sets of blood cultures

Two sets of blood cultures -key to making a diagnosis is blood cultures

A client is scheduled for an intravenous pyelogram (IVP). Which information from the client's history indicates the greatest potential hazard for this test? Type 2 diabetic taking metformin (Glucophage) Constipation Urge incontinence Hypertension

Type 2 diabetic taking metformin (Glucophage) -those w/ diabetes @ greater risk of developing kidney failure following administration of the dye -metformin can cause additional problems if kidney failure occurs

The nurse is reviewing the assessment data of a client suspected of having diabetes insipidus. Which of these findings should the nurse expect after a water deprivation test? Rapid protein excretion Decreased serum potassium Unchanged urine specific gravity Increased edema and weight gain

Unchanged urine specific gravity -even when fluids restricted, pt continues to excrete large amounts of dilute urine DI = kidneys unable to conserve water

A nurse needs to administer cardiopulmonary resuscitation to a 5 year-old child. In order to be effective, the nurse should take which action as a single rescuer? Assess the brachial pulses bilaterally Initiate compression-only CPR and compress the chest at least 1 1/2 inches Use a ratio of 2 breaths to 30 compressions Compress the chest at a rate of about 90 times per minute

Use a ratio of 2 breaths to 30 compressions -compression rate @ least 100 bpm -compression-only CPR recommended for lay persons -compressions should be 1/3-1/2 of chest depth in kids

The client has undergone a dilation and curettage (D & C) following a spontaneous abortion at 8 weeks. To promote an optimal recovery, what information should the nurse include in the discharge teaching? (Select all that apply.) Expect heavy bleeding for at least a week Use sanitary pads until vaginal bleeding has stopped Strenuous sport activities should be postponed until bleeding stops Resume vaginal intercourse 6 weeks after the procedure Referral for grief counseling

Use sanitary pads until vaginal bleeding has stopped strenuous sport activities should be postponed until bleeding stops referral for grief counseling -client should avoid vag intercourse + not to use tampons for ~2 weeks

A woman in labor calls a nurse to assist her in the bathroom. The nurse notices a large amount of clear fluid on the bed linens. The nurse should act based on knowledge that fetal monitoring must now assess for what complication? Variable decelerations Early decelerations Late accelerations Periodic accelerations

Variable decelerations -when the membranes rupture, there is increased risk initially of cord prolapse if the head is @ a minus level

The nurse prepares to give a 1 year-old child an intramuscular injection. At what site should the nurse administer the injection? Ventrogluteal Incorrect Deltoid Vastus lateralis Dorso gluteal

Vastus lateralis -lies along lateral aspect of thigh -this site able to tolerate larger volumes -not located near any nerves or bv's

A client is admitted with the diagnosis of pulmonary embolism (PE). While taking a history, the client says: "I was admitted for the same thing twice in the past six months. In fact, the last time was just three months ago." The nurse should anticipate a need for education may be centered around which approach to treatment? Vena cava filter Pulmonary embolectomy Thrombolytic therapy Increase in the anticoagulant dosage

Vena cava filter -pt's w/ recurrent PE or those w/ excessive clotting complications r/t medical therapy may require vena cava interruption -this is the placement of a filter device in the inferior vena cava - traps clots before they reach the pulmonary circulation

The nurse is auscultating the heart of a client who is diagnosed with dilated cardiomyopathy. What finding would the nurse expect to hear? Split S2 Diastolic murmur Apical click Ventricular gallop of S3

Ventricular gallop of S3 -caused by blood flowing rapidly into a distended noncompliant ventricle -most common sound w/ L sided heart failure

The nurse is caring for two children who have had surgical repair of congenital heart defects. For which defect is it the highest priority to assess for findings of heart conduction disturbance? Patent ductus arteriosus Aortic stenosis Ventricular septal defect Atrial septal defect

Ventricular septal defect

A nurse experiences a needle stick with a used hypodermic needle. What action should the nurse perform immediately? Vigorously wash the affected area with soap and water Contact employee health services Look up the policy on needle sticks Notify the supervisor and risk management

Vigorously wash the affected area with soap and water -will help remove possible contamination

The client is admitted to the emergency department with hypertensive crisis. Which finding requires immediate action by a nurse? Crackles at the lung bases Lower extremity pitting edema Weakness in left arm Jugular vein distension

Weakness in left arm -in a pt w/ uncontrolled HTN, weakness in the extremities is a sign of cerebral involvement w/ risk for stroke remaining 3 r/t fluid overload

A nurse is caring for a 10 year-old child who will be started on heparin therapy. Which assessment is critical for the nurse to make before initiating this therapy? Skin turgor Vital signs Lung sounds A nurse is caring for a 10 year-old child who will be started on heparin therapy. Which assessment is critical for the nurse to make before initiating this therapy? Skin turgor Vital signs Lung sounds Weight

Weight -b/c dosage for anticoags in kids is calculated on basis of wt

The nurse is caring for a client with a colostomy pouch. During a teaching session, when should the nurse teach that the pouch should be emptied? Prior to meals After each fecal elimination When it is one-third to one-half full At the same time each day

When it is one-third to one-half full -if the pouch becomes more than 1/2 full it may separate from the flange -@ same time each day does not account for the pouch being empty @ that time

A nurse is assessing the growth of children during their school-age years. What would the nurse expect to see during this assessment? Progressive height increase of 4 inches each year Decreasing amounts of body fat and muscle mass Yearly weight gain of about 5 1/2 pounds per year Little change in body appearance from year to year

Yearly weight gain of about 5 1/2 pounds per year -increase of 2 inches in ht

Central diabetes insipidus may be caused by damage to the

hypothalamus

pt's w/ SIADH should be placed on

seizure precautions -hypersecretion of ADH causes hyponatremia -as serum sodium drops, extra water enters body cells + causes them to swell -convulsions, shock, coma, death may occur


संबंधित स्टडी सेट्स

Management Practices (Sheep, Swine, and Goats)

View Set

Chapter 5: Devices and Infrastructure 5.1-5.8

View Set

Four Basic Styles of Communication

View Set

Français Unité 1: La France et Sa Capitale Questions

View Set

Chapter 1 basic insurance concepts and principles

View Set

Legal and Ethical Issues for health professions Final

View Set

Business Intelligence MIS 5342 Baylor

View Set